Surgery Sixer For NBE
Surgery Sixer For NBE
2020-2019
RECENT PATTERN QUESTIONS 2020 Section B: Head and Neck, Thyroid and Breast
a. PMMC flap
b. DIEP flap
c. LD flap
d. DP flap
12. Which type of Cancer breast has presence of Mucin on
Biopsy?
a. Battle sign b. Milian’s ear sign
a. Medullary b. Tubular
c. Injury to pinna d. Erysipelas
c. Colloid d. Basal type
XXII
Latest Questions Paper 2020-2019
Section C: GastroIntestinal Tract 17. What is your diagnosis from the Barium enema image
shown below?
13. A patient aged 30 years presents with Dysphagia. High
resolution manometry was done for that patient showed the
following findings. There was Panesophageal pressurisation
with Distal Contractile integrity as >450 mm Hg cm. What
is the diagnosis?
a. Type 1 achalasia
b. Type 2 achalasia
c. Type 3 achalasia
d. Diffuse esophageal spasm
14. A 25-year-male after a full stomach meal presents with
abdominal pain following a sudden vomiting. On X-ray there
is Pneumo mediastinum with Widening of Mediastinum.
What is the most probable diagnosis?
a. Rupture of emphysematous bulla
b. Perforated peptic ulcer a. Intussusception b. Sigmoid Diverticulum
c. Spontaneous rupture of esophagus c. Cancer colon d. Polyp
d. Foreign body in esophagus
18. A patient presents with Purulent peritonitis and laparotomy
15. A patient presents with halitosis and swelling in the left side was done, there was a diverticulum in the sigmoid colon
of neck. On pressing gives gurgling sound. Barium image is which is perforated. What is the stage of this Diverticulitis?
shown below. What is your diagnosis? a. Hinchey’s Stage 1 b. Hinchey’s Stage 2
c. Hinchey’s Stage 3 d. Hinchey’s stage 4
19. A 20-year-male underwent a major accident and injury in
abdomen. For him the entire ileum and part of the jejunum
is resected. Which of the following problem the patient will
suffer from?
a. Vitamin B12 deficiency
b. Constipation
c. Gastric Ulcer
d. Bleeding in GIT
20. Patient underwent LSCS- postoperatively patient developed
abdominal distension and obstipation. Examination showed
absence of bowel sounds and Soft consistency of palpation.
What is your diagnosis?
a. Zenker’s diverticulum b. Laryngocele a. Uterine Rupture
c. Dysphagia lusoria d. Schatzki ring b. Paralytic ileus
c. Appendicitis
16. Identify the Barium swallow image shown below:
d. Amniotic fluid peritonitis
21. Hirschsprung disease is due to
a. Atrophy of Intestinal Villi
b. Crypt abscess formation
c. Failure of Neural crest Cells to migrate
d. Failure of fusion of Cloaca
23. Which is the most common pancreatic endocrine neoplasm? Section E: Specialty Surgeries
a. Insulinoma b. VIPoma
c. Glucagonoma d. Gastrinoma Vascular Surgery
24. What is the most common site of gastrinoma in MEN1
29. A smoker patient presents with abdominal aortic aneurysm
syndrome?
with diameter of 44 mm with non-specific abdominal pain
a. Jejunum b. Ileum
what is the ideal answer below?
c. Duodenum d. Stomach
a. Immediately operate
25. A 5 years old male child with acute fulminant liver failure. b. Operate when size becomes >55 mm diameter
Which one of the following criteria are not included in the c. Serial USG advised until it becomes 77 mm diameter
KING’s college criteria for Liver transplant? d. No treatment needed.
a. Age <10 years
30. A 70-year-old male smoker comes to hospital with
b. Prothrombin time >50 seconds
intermittent claudication pain in both the calf and in the
c. Bilirubin >300 Micromol/litre
Buttock claudication is present. What is the most common
d. Jaundice less than 7 days before development of
site of Atherosclerosis?
encephalopathy
a. Aortoiliac obstruction b. Femoral obstruction
26. What is the Modified Child Pugh class for patient who c. Iliac obstruction d. Distal obstruction
has a serum bilirubin of 2.5 mg/dl, Serum albumin 3 g/dl,
31. True statement about intermittent claudication
Prothrombin time increased 5 seconds. Moderate ascites is
a. Felt at rest
present but there is Grade 2 encephalopathy.
b. Most common site is the calf
a. CP A
c. Claudication distance cannot vary from day to day
b. CP B
d. Relieved after getting out of bed and walking
c. CP C
d. CP D 32. The sign seen on USG when Common femoral vein meets
the GSV and Femoral artery at the saphenofemoral junction
27. A patient diagnosed as Carcinoma Uncinate process
is called as
presents with sudden abdominal pain on one day. The
a. String sign b. Saphenous eye
tumor has infiltrated into
c. Mickey mouse sign d. Stemmer sign
a. Portal vein
b. SMV 33. Identify the cause for the swollen limb shown below
c. SMA
d. Splenic vein
28. A 35-year-old male presents with recurrent episodes of fever
with chills and rigors. On examination Jaundice is seen.
MRCP image is shown below. What is your diagnosis?
XXIV
Latest Questions Paper 2020-2019
Urology 41. What is the T staging of a lung tumor than 2.5 cm in Size as
per AJCC classification?
35. Management of a renal cell cancer size <4 cm is a. T1a b. T1b
a. Radical nephrectomy c. T1c d. T2
b. Partial nephrectomy
42. What is the most common infection in a post renal
c. Chemotherapy
transplant patient after 3 to 4 months?
d. Surgery followed by chemotherapy
a. Bacterial b. CMV
36. A man is brought to the emergency after he fell into a c. BK virus d. EB virus
man-hole and injured his perineum. He feels the urge to
43. Which of the following is not a component of Thoracoscore?
micturate but is unable to pass urine and there is blood at
a. Performance status
the tip of the meatus with extensive swelling of the penis and
b. ASA Grading
scrotum. What is the location of injury?
c. Complication of Surgery
a. Bulbar urethra b. Prostatic urethra
d. Priority of Surgery
c. Bladder d. Membranous urethra
44. A middle-aged man with a swelling over neck since
37. Identify the pathology in the image shown here:
childhood, neck swelling has a bag of worm appearance.
Bruit heard over the swelling. What is your diagnosis?
a. Pituitary adenoma
b. Craniopharyngioma
c. Meningioma
d. Oligodendroglioma
a. Omphalocele b. Gastroschisis
c. Umbilical hernia d. Spigelian hernia
XXV
Surgery Sixer for NBE
46. A 45-year-old male had sudden severe headache and sudden 52. The score used for Wound infection for surveillance
neck rigidity. The CT image shown below. What is your (AIIMS May 2019)
diagnosis? a. Southampton scoring system
b. Glasgow
c. SIRS score
d. APACHE score
53. All the following are included in SIRS definition Except:
(AIIMS May 2019)
a. Temperature >38°
b. Heart rate >90 beats/min
c. Respiratory rate <20 breaths/min
d. WBC Count >12000 or <4000
54. What is the percentage of Surgical Site infection in Clean
Contaminated wound with antibiotic prophylaxis?
(AIIMS May 2019)
a. 1% b. 3%
c. 6% d. 7%
a. Subarachnoid hemorrhage 55. Following RTA, a patient had multiple rib fractures, BP-
b. Meningitis 90/60 mm Hg, Tachypnea, Bluish Discoloration seen in
c. Haemorrhagic stroke Chest wall. There is absence of Breath sounds in Right Side.
d. SDH What will you do next? (AIIMS May 2019)
47. An elderly female with a long standing mole in face for a. Wide bore needle insertion in 2nd ICS
many years presents with irregular border, enlarging size of b. Rush to operation theatre for emergency thoracotomy
the mole suddenly. What is the cause? c. Take X-ray chest PA view
a. Superficial spreading type d. Intubate the patient
b. Lentigo maligna melanoma 56. To prevent contamination, arrange the sequential order of
c. Acral lentiginous removal of Personal protective Equipment after surgery.
d. Nodular melanoma (AIIMS May 2019)
48. In a patient with Frost Bite what is the temperature of water • Glove
that must be used to recover the patient: • Apron
a. 37° • Face shield
b. 42° • Mask
c. 48° Order:
d. 52° a. Glove- Face shield- Gown- Mask
49. A 40-year-man with diagnosis of ADPKD dependent on b. Mask- Face shield- Gown- Glove
Tolvaptan now presents with abdominal pain. What is the c. Mask- Glove- Face shield- Gown
cause? d. Gown- Glove- Face shield- Mask
a. Colonic perforation
57. On Massive blood transfusion the following complication
b. Side effects of tolvaptan
occurs. (AIIMS November 2019)
c. Colonic diverticulitis
a. Respiratory acidosis b. Metabolic acidosis
d. Appendicitis
c. Metabolic alkalosis d. Respiratory alkalosis
50. A graft taken from an Identical Twin is
58. Identify the drain shown below (AIIMS November 2019)
a. Autograft b. Allograft
c. Isograft d. Xenograft
Section A: General
XXVI
Latest Questions Paper 2020-2019
59. Identify the image shown below? (AIIMS November 2019) 60. What is the name of the therapy given in the image below?
(AIIMS November 2019)
a. Bipolar cautery
a. Negative pressure healing
b. Monopolar cautery
b. Positive pressure healing
c. Harmonic scalpel
c. Sclerotherapy
d. LigaSure
d. Cryotherapy
A B C D
63. What is the Stage of Bedsore which is having sacrum exposed as shown below? (AIIMS November 2019)
XXVII
Surgery Sixer for NBE
64. The Cannula used to infuse maximum fluids in Dehydration 71. Video Question on thyroid Examination- Name the test
and Diarrhea is (AIIMS November 2019) (AIIMS November 2019)
a. Grey b. Green
For video, scan this QR Code
c. Pink d. Blue
a. Lahey’s test
b. Pemberton test
a. Prevents viral infections
c. Pizzillo test
b. Prevents transfusion-related reactions
d. Crile test
c. Prevents infection
d. All of the above 72. A 25-years-old male complaints of pain in the right side of
forearm and arm- the following test is done to demonstrate
the diagnosis. (AIIMS November 2019)
Section B: Head and Neck, Thyroid and Breast
For video, scan this QR Code
66. Post-Superficial parotidectomy a patient developed numb-
ness over the cheek area. Which nerve is injured in the sur-
gery? (AIIMS May 2019 and November 2019)
a. Auriculotemporal nerve
b. Greater auricular nerve
c. Mandibular nerve
d. Facial nerve
67. Which of the following is not removed in Radical Neck
Dissection? (AIIMS May and November 2019)
a. Level 2 nodes b. Submandibular gland
c. Sublingual gland d. Tail of parotid gland
68. Thyrotoxic Crisis in Post op patient is due to
(AIIMS May 2019) a. Roos Test b. Pemberton test
a. Preop inadequate preparation c. Allen test d. Wright test
b. Rough handling during surgery 73. Identify the name of the disease shown here in the Breast?
c. Parathyroid removal (AIIMS May 2019)
d. Anaesthetic drug induced.
69. Which of the following is true about thyroglossal cyst?
(AIIMS May 2019)
a. Secondary infection leads to sinus.
b. 70% contains heterotopic thyroid tissue
c. Resolve spontaneously in 20-30% cases.
d. Thyroglossal duct obliterates by 10th week of IUL
e. Lining epithelium is stratified squamous epithelium or
Ciliated Columnar Epithelium
70. The following are locally advanced Breast cancer except:
(AIIMS May 2019)
a. Size of tumour 4 cm a. Mondor’s disease
b. Chest wall fixity b. Zuska disease
c. Skin involvement c. Duct ectasia
d. Inflammatory breast cancer d. Phyllodes
XXVIII
Latest Questions Paper 2020-2019
XXIX
Surgery Sixer for NBE
89. A 25-year-old college student met with road traffic accident Section B: Head and Neck, Thyroid, Breast
and is admitted in casualty. He has marked abdominal
distension, a pulse rate of 140 and a BP of 80/50 mm Hg. 96. Shrugging of shoulder following neck surgery due to injury
The most appropriate initial investigation would be to (JIPMER May 2019)
(JIPMER May 2019) a. Spinal Accessory Nerve
a. Abdominal paracentesis b. Thoracodorsal Nerve
b. CT abdomen c. Bell’s Nerve
c. Plain X-ray of abdomen d. Vagus Nerve
d. FAST scan 97. Mr. Ramu, a 30-years-old male with Papillary carcinoma in
90. A Patient with Necrotizing fasciitis is treated with antibiotics Thyroid with a Nodule <3 cm confined to neck with 2 lymph
and debridement. He got discharged on wound recovery. nodes palpable in neck with Lung micro metastasis present.
Now he came back after 10 days with diarrhea and fever. The Stage according to AJCC 8th Edition is (JIPMER May 2019)
following are done except (JIPMER May 2019) a. I
a. Continue same antibiotic b. II
b. Stop Loperamide 2 mg c. III
c. Metronidazole is the Drug of Choice d. IV
d. Washing hands with alcohol-based hand sanitizers are 98. Which is true regarding breast cancer in pregnancy
superior to washing hands with soap to stop the spread of (JIPMER May 2019)
diarrhea a. Mammography is a diagnostic modality in breast cancer
91. A 24-year burns patient weighing 60 kg presents with b. Easily diagnosed in pregnant females than that of non
involvement of face, neck, anterior chest wall and abdominal pregnant females
wall and both arms circumferential burns. Calculate the c. Most tumors are hormone receptor negative
fluid to be given in the first 8 hours d. Ductal carcinoma is most common type
(JIPMER December 2019 Pattern) 99. All are true regarding inferior pedicle breast surgery except
a. 5.4 L b. 6.5 L (JIPMER May 2019)
c. 7 L d. 10.8 L a. It is a technique used for reduction mammoplasty
92. A 4-years-old child presented with burns. The burns seen b. This procedure is contraindicated in smoking females
in head and neck fully, right upper limb fully except palm c. Lactation is possible always after the surgery
and Right lower limb fully including foot. What is the d. The removed Breast tissue is sent for pathological study
percentage of burns? (JIPMER December 2019 Pattern) always.
a. 14 b. 72 100. Long acting thyroid stimulator (LATS) is responsible for
c. 41 d. 24 which of the following disease?
93. Suture used for bile duct surgery (JIPMER December 2019 Pattern)
(JIPMER December 2019 Pattern) a. Papillary Ca of Thyroid
a. Vicryl b. Hashimoto’s disease
b. Catgut c. Grave’s disease
c. Polyprophylene d. MNG disease
d. Silk 101. A 36-year-old woman presents with thyroid swelling.
94. A patient is admitted to emergency department with head Radio-iodine uptake scan showed diffuse uptake. Which
injury. He opens eyes to painful stimulus, localizes pain intervention would benefit the patient best?
and produces Incomprehensible sounds. What would be his (JIPMER December 2019 Pattern)
Glasgow coma scale? (JIPMER December 2019 Pattern) a. Hemithyroidectomy
a. 13 b. 9 b. Radioactive iodine
c. 7 d. 6 c. No treatment
d. Anti-thyroid drug
95. According to ATLS guideline, in a patient with paraplegia
102. Cervical rib passes between
‘Hard neck collar’ is indicated in
(JIPMER December 2019 Pattern)
(JIPMER December 2019 Pattern)
a. Scalenus anterior and medius
a. All trauma patients
b. Scalenus medius and posterior
b. Presence of neck pain
c. Clavicle and 1st rib
c. History of neck injury
d. Scalenus posterior and vertebra
d. Neurological deficit present
XXX
Latest Questions Paper 2020-2019
XXXI
Surgery Sixer for NBE
116. Examine the schematic diagram of hydrocele carefully. 123. 0.5 cm follicular neoplasm with no LN involvement,
Which of the following types does this fit in? management
(JIPMER December 2019 Pattern) a. Lobectomy
b. Hemithyroidectomy
c. Total thyroidectomy
d. Total thyroidectomy with lymphadenectomy
e. Total thyroidectomy with radioiodine ablation
124. Burns percentage is calculated by
a. Palm rule b. Lund and browder
c. Wallace rule d. Berkow Formula
e. Parkland formula
125. Distributive Shock types are
a. Sepsis b. Neurogenic
a. Congenital hydrocele c. Anaphylaxis d. Haemorrhagic
b. Vaginal hydrocele e. Cardiogenic shock
c. Infantile hydrocele
d. Hydrocele of cord 126. Weigert Meyer applicable for
a. Duplication of ureter b. Ectopic ureter
117. A 28-year-old male presents with sudden detumescence, c. VUR d. Horse shoe kidney
swelling and pain in penis following a loud pop sound e. PCKD
during sexual intercourse. What of the following is true?
(JIPMER December 2019 Pattern) 127. Direct Hernia- True statement is
a. Tear of tunica vaginalis a. After reduction comes in the same spot
b. Rupture of buck’s fascia leads to bleeding limited only to b. Defect felt below and lateral to pubic tubercle
penis (intrapenile hematoma) c. Common in Children and Adults
c. Immediate surgical exploration and repair d. Descend through inguinal Canal
d. Urethral injury is always associated e. MC in men
118. False statement regarding Peyronie’s disease 128. True regarding pancreatic pseudocyst
(JIPMER December 2019 Pattern) a. Distinct upper border can be felt
a. Immediate surgery during active stage gives best results b. Fluctuant
b. Dorsal deviation is most common c. Moves poorly with respiration
c. Associated with Dupuytren’s contracture d. Fluid thrill present
d. Nesbitt procedure is done. 129. Lymphoma staging is
119. All are radio opaque stone except a. Ann Arbor staging b. Cotswold staging
(JIPMER December 2019 Pattern) c. Lugano staging d. Nesbitt staging
a. Uric acid b. Oxalate e. Gleason staging
c. Cystine d. Staghorn 130. A 19-year-old boy presents with acute right testicular pain,
causes
a. Varicocele b. Torsion
PGI NOVEMBER 2019 c. Epididymo orchitis d. Testicular tumor
120. True statement about Bjork flap is/are: 131. Regarding Testicular tumors – True statement are:
a. Allows re introduction of Displaced tube a. Seminomas spread via lymphatics
b. A type of tracheostomy flap b. Lymphatic drainage is to Para aortic nodes.
c. Avoid 1st tracheal ring c. Inguinal nodes are involved.
d. Free edge of the Bjork flap is sutured to the Transverse skin d. Tumor markers are elevated
incision e. Transscrotal orchiectomy is done
e. This is a superiorly based flap 132. Sigmoid volvulus which are true
121. True about diaphragmatic injury is a. Rectal Decompression
a. Can occur after blunt trauma to lower chest b. Pelvic Colon was rotated against adhesion
b. Should always be operated by thoracotomy c. Thumb printing sign
c. CT is more sensitive in diagnosing it d. Show anticlockwise rotation
d. The defect is mostly large enough to cause symptoms 133. Parasites causing cholangio carcinoma
e. Penetrating injury below 5th Intercostal space can cause a. Opisthorchis viverrini b. Clonorchis sinensis
diaphragmatic injury c. Streptococcus d. Schistosoma
122. In acute cholecystitis which of the following sign you see e. E. coli
a. Murphy sign b. Boas sign 134. Anastomosis of intestines with different diameters is by
c. Pointing sign d. Obturator sign a. Cheatle cut b. End to side
c. Side to side d. End to end
XXXII
Latest Questions Paper 2020-2019
140. A patient is with Courvoisier positive is examined for 149. True statement(s) regarding Gall stone ileus is/are:
jaundice, which of the following can be seen a. Caused by mechanical blockage by impacted stones
a. Tenderness over gallbladder area b. Also called Mirizzi syndrome
b. Palpable of gallbladder c. Functional block by irritation of the stone
c. Peripancreatic carcinoma d. Associated with enterobiliary fistula
d. Obstruction in common bile duct e. Treatment require surgery in all cases
e. Acalculous cholecystitis. 150. Which of the following is/are true statement(s) regarding
141. SVC syndrome is caused by pre-operative antibiotic prophylaxis:
a. Thymoma a. Should be given according to sensitivity of the spectrum of
b. Pancoast tumor of right lung apex organisms expected to be encountered during the surgery
c. Oesophagus growth b. Should be started 1-2 hour before the induction of
d. Coarctation of aorta anesthesia
e. Retrosternal goitre c. Maximum blood and tissue concentration of antibiotic
should reach when handling the organ of surgical interest
142. Branchial cyst/fistula – True statement d. Should be repeated if surgery is prolonged
a. Other name cystic hygroma e. Should ideally be continued post-operatively
b. Due to 2nd brachial arch remnant
c. Always unilateral 151. Treatment of intermediate risk prostate cancer include(s):
d. Transillumination may be seen a. Radical prostatectomy b. Chemotherapy
e. In cases of sinus inner tract is at tonsillar fossa c. Radical radiotherapy d. Orchidectomy
e. Medical castration
143. Postoperative Ileus- True statement
a. Mostly require surgery and immediate re exploration 152. There is involvement of one lobe (on rectal examination)
b. Small intestine recover faster than colon with no capsular involvement and ipsilateral lymph node
c. Stomach recover faster than small intestine by tumor. Which of the following stage of Prostate cancer
d. Vomiting and abdominal distension seen staging system best depicts this picture:
e. MC cause is electrolyte imbalance a. T1 b. T2a
c. T3 d. N1
e. N2
XXXIII
Surgery Sixer for NBE
153. True about Lanz incision for appendectomy: 163. True about Ectopic ureter:
a. Transverse skin crease incision a. Almost always associated with double ureter
b. Oblique muscle cutting incision b. Bilateral more common
c. Centered on Mc Burney point c. Drain lower pole of kidney
d. Centered on the midclavicular–mid-inguinal line d. Open distally beyond the normal opening
e. Midline incision of the abdomen just below the umbilicus e. Incontinence of urine is more common in males than
154. Which of the following is/are presenting symptom(s) of Left females
colon carcinoma: 164. Wrong statement(s) about lower limb varicose veins:
a. Hematochezia b. Melena a. Varicosities on posterolateral aspect of leg is due to
c. Anemia d. Intestinal obstruction accessory saphenous vein
e. Altered bowel habits b. Varicosities on posterolateral aspect of leg is due to
155. Colorectal cancer involving local serosa and two localized blockade of short saphenous vein
lymph nodes will come under the following stage: c. Varicosities on medial aspect of upper part of thigh is due
a. T2 b. T3 to blockade of great saphenous vein
c. T4 d. N1 d. Anterolateral side varicosities are due to blockade in
e. N2 accessory saphenous vein
156. True statement(s) about rectal procidentia is/are: 165. Which of the following is/are true regarding deep venous
a. Most commonly seen in elderly men thrombosis :
b. Better seen in squatting position a. Flight journey more than 6 hours is a risk factor
c. Kegel exercises is helpful in prolapse b. Usually bilateral swelling
d. Surgery is best treatment c. Tenderness can be elicited in calf muscles on applying
pressure
157. Unlike an ileostomy, a colostomy is:
d. Low grade fever with increased pulse rate
a. A temporary procedure
e. Affected vein can be seen like cord
b. It is brought out in the right iliac fossa
c. Flushing is done on the abdominal surface 166. True statement(s) about mesh surgery for hernia:
d. Effluent is usually solid a. Absorbable meshes are most commonly used
e. Permanent procedure b. Mesh can be braided
c. Mesh can be attached to defect by glue or suture or staples
158. PAIR treatment for hydatid cyst include(s):
d. Suturing mesh edge to edge into the defect with no overlap
a. Cystectomy b. Aspiration
is recommended
c. Puncture d. Injection
e. Shrinkage of mesh may occur after some years
e. Anesthesia
167. Which of the following local flap(s) can be used on convex
159. Risk factor(s) for adenocarcinoma of esophagus is/are:
surface:
a. Smoking b. Alcohol
a. Advancement flap b. V-Y Advancement Flap
c. GERD d. Barrett’s esophagus
c. Rotation flap d. Bipedicle flap
e. Obesity
e. Bilobed flap
160. True about Annular pancreas:
168. Unlike port wine, salmon patch is:
a. Commonly seen in Cystic fibrosis
a. Resolves spontaneously
b. Commonly associated with Downs syndrome
b. Reddish-purple in color
c. Obstruction of first part of duodenum
c. Can occur later in life
d. It needs no treatment
d. Due to dermal capillary malformation
e. Treated by surgery
e. Usually disappears at 1yr of age
161. Which of the following is/are cause of toxic megacolon:
169. As compared to Billroth type II gastrostomy, Billroth type I
a. Hirschsprung’s disease
gastrostomy is characterized by:
b. Cystic fibrosis
a. Anastomosis b/w stomach and jejunum
c. Inflammatory bowel disease
b. Anastomosis b/w stomach and 1st part of duodenum
d. Colitis by clostridium difficile
c. Anastomosis b/w stomach and 2nd/ 3rd part of duodenum
e. Radiation colitis
d. Closure of duodenum
162. True statement(s) about Anal fissures: e. Roux-en-Y anastomosis
a. Most commonly seen posteriorly in the midline
170. Early sign of compartment syndrome is/are:
b. Commonly due to local trauma, vaginal delivery, anal sex
a. Pain b. Feeble or absent pulse
and constipation
c. Dry gangrene d. Paresthesia
c. Lateral internal sphincterotomy is the surgical procedure
e. Pallor
of choice
d. Extends proximal to the dentate line 171. Feature(s) of Melanoma include(s) all except:
e. Conservative management include dietary modifications a. No color change b. Asymmetric outline
c. Elevation d. Changing irregular borders
e. Diameter less than 5 mm
XXXIV
Latest Questions Paper 2020-2019
172. Which of the following statement(s) is/are correct regarding 180. Which Instrument is shown below?
Abdominal laparoscopy surgery:
a. Single incision technique uses umbilical incision for trocar
insertion
b. CO2 used in most cases
c. Trocar is placed at 90 degree to abdominal wall
d. Pressure up to 30mm Hg is maintained throughout surgery
e. Verres needle is useful for creating pneumoperitoneum
FMGE 2019
a. Artery forceps
Section A: General Surgery and Trauma b. Kocher forceps
c. Allis forceps
173. A patient presented to emergency after RTA with multiple d. Babcock forceps
rib fractures. He is conscious, speaking single words. On
examination, respiratory rate was 40/min and BP was 90/40 181. Comment on the diagnosis?
mm Hg. What is the immediate next step?
a. Urgent IV fluid administration
b. Intubate the patient
c. Chest –ray
d. Insert needle in 2nd intercostal space
174. Correct procedure of inserting Nasogastric tube is.
a. Supine with neck flexed
b. Supine with neck extended
c. Sitting with neck flexed
d. Sitting with neck extended
175. Calculate the GCS of a patient exhibiting eye opening
on pain, conscious but confused and cannot tell time and
exhibits flexion on painful noxious stimuli to the arm.
a. 8 a. Sebaceous cyst
b. 9 b. Implantation dermoid
c. 10 c. Angular dermoid
d. 11 d. Lipoma
176. Most appropriate management of recurrent keloid is? 182. Comment on the diagnosis
a. Excisional surgery
b. Intramarginal excision followed by radiation
c. Cryosurgery
d. Silicone gel sheeting
177. Seat Belt injury leads to?
a. Splenic laceration
b. Splenic contusion
c. Gut ischemia
d. Mesenteric adenitis
178. Which of the following is not scanned by FAST USG?
a. Pericardium
b. Pleural cavity
c. Spleen
d. Liver a. Lipoma
179. Prophylactic antibiotics to minimise SSI are b. Encephalocele
given_____________? c. Cystic hygroma
a. 60 minutes before skin incision d. Lymphadenopathy
b. 1-3 hours before skin incision 183. In hypovolemic shock which organ should be assessed for
c. At time of surgical incision determining under-perfusion?
d. Night before surgery for peaking of effect a. Kidney b. Heart
c. Lung d. Liver
XXXV
Surgery Sixer for NBE
184. A lady with 50% burns of dermis and subcutaneous 192. A man was presented to emergency department with head
tissue came to emergency department. Burns will be injury after an accident with vehicle. Investigation of choice
Classified as; should be;
a. 1st degree a. CECT
b. 2nd degree superficial b. MRI
c. 2nd degree deep c. NCCT
d. 3rd degree burn d. MRI
185. Cellulitis is
a. Nonsuppurative and noninvasive
Section B: Head and Neck, Thyroid and Breast
b. Suppurative and noninvasive
c. Nonsuppurative and invasive 193. A 27-year-old woman presents with 26 weeks of gestation
d. Suppurative and invasive with a thyroid lesion which is found to be papillary
186. 40 years old patient with femur fracture with pulmonary carcinoma of thyroid. Which is the best treatment for this
infiltration and respiratory distress patient?
a. Fat embolism a. Thyroid ablation using radioactive Iodine
b. Pulmonary embolism b. Total thyroidectomy
c. Air embolism c. Observation
d. Obstruction d. Hemi-thyroidectomy
187. 32-year man presented with fever and pain in upper right 194. Which procedure is done in case of Ranula management?
hypochondrium after food intake. Investigation of choice; a. Incision and drainage b. Aspiration
a. CECT c. Excision d. Sclerosant injection
b. Ultrasound 195. What is the location of Killian dehiscence?
c. MRI a. Below Superior constrictor
d. HRCT b. Below Inferior constrictor
188. Unconscious patient, most common cause of respiratory c. Below cricopharyngeal muscle
obstruction; d. Below upper 1/3rd of smooth muscle of oesophagus
a. Tongue fall 196. Most common oral cancer;
b. Head injury a. Squamous cell ca b. Adenocarcinoma
c. Respiratory centre block c. Transition cell ca d. Mucoepidermoid
d. Hyoid fracture
197. Which patient has better prognosis in breast cancer?
189. In an accident case, after the arrival of medical team, all a. Luminal A
should be done in early management except; b. Luminal B
a. Stabilization of cervical vertebrae c. Patient who has triple test negative
b. Check BP and Pulse d. Patient who has triple test positive
c. Check Respiration
d. Glasgow coma scale
190. Patients are categorized on the basis of chances of survival Section C: GIT
in disaster management is;
198. A 7-day old infant presents with bilious vomiting and gross
a. Triage b. Mitigation
abdominal distention with absent bowel sounds. X-ray
c. Tagging d. Surge capacity
abdomen shows multiple gas filled loops. Diagnosis is?
191. Long term diabetic PATEINT with blisters walked barefoot a. Hirschsprung disease
few miles on hot sand. He presented with this clinical b. Congenital Hypertrophic pyloric stenosis
condition. What is the most probable diagnosis? c. Duodenal atresia
d. Malrotation of gut
199. Meconium ileus is a presentation seen in which of the
following diseases?
a. Mucoviscidosis b. Hirschsprung disease
c. Ileal atresia d. Congenital aganglionosis
200. Which is the first investigation to be done in case of Neonate
presenting with frothiness in mouth and dyspnoea?
a. Bronchoscopy with injection of methylene blue
b. NG Tube insertion and CXR to check position of tube
c. CT chest
a. Diabetic foot
d. Endoscopy
b. Burn
c. Necrotizing fasciitis
d. Cellulitis
XXXVI
Latest Questions Paper 2020-2019
201. Best treatment strategy for Anal cancer? 206. Emergency Surgery is indicated in Ulcerative Colitis in all
a. Chemoradiation except?
b. Surgery a. Toxic megacolon b. Colonic perforation
c. Radiation c. Colonic obstruction d. Refractory fistula
d. Chemotherapy 207. 20-years-old female diagnosed previously with adenocarci-
202. Which is false about Crohn’s disease? noma stomach and on examination following is seen;
a. No occurrence after surgery
b. Aphthous ulcer
c. Skip lesions
d. Fistula formation
203. Which of the following is not done in Carcinoma
oesophagus?
a. Biopsy
b. pH-metry
c. CT chest
d. PET scan a. Sister marry joseph nodule
204. A 30-year-old man presents with a four days history of right b. Ulcer
iliac fossa pain. USG image is shown below. Which is the c. Infected umbilical hernia
best management algorithm? d. Irish node
208. Distal ileum was removed in a 20-year-old girl. Which
absorption deficient will be seen?
a. Iron b. Bile salts
c. Folic acid d. Copper
209. 28 year old alcoholic patient walks to hospital with the
complaints of binge vomiting, chest pain, fever and
pneumomediastinum. The most probable condition;
a. Boerhaave syndrome b. Tension pneumo thorax
c. PUD perforation d. Mallory-Weiss tear
210. True about Crohn’s disease except
a. Recurrence is more common
b. Rectum is involved
c. Fissures are formed
a. Ochsner-Sherren regimen d. Transmural
b. Urgent appendicectomy 211. Patient underwent gastrectomy, after eating within 20
c. Extraperitoneal drainage and parenteral antibiotics minutes sweating, diarrhoea, what could be the cause?
d. Per cutaneous drainage and parenteral antibiotics a. Hyperglycemia b. Early dumping syndrome
205. Comment on the diagnosis of a film shown of a 65-year-old c. Late dumping syndrome d. Hypoglycemia
man with acute abdomen 212. Risk factors of oesophageal carcinoma are all except
a. GERD b. Betel chewing
c. Nitrates food d. Anti oxidant deficiency
213. ALVARADO score 2 defines:
a. Temperature
b. Leukocytosis
c. Tenderness in left iliac fossa
d. Migratory pain
a. Testicular torsion
b. Hydrocele
c. Varicocele a. Extradural hemorrhage
d. Testicular atrophy b. Subdural hemorrhage
c. Subarachnoid hemorrhage
222. Rodent ulcer is
d. Intracerebral hemorrhage
a. Basal cell cancer b. Squamous cell ca
c. Rhinophyma d. Adenocarcinoma 230. Thimble bladder is seen in:
a. Cancer b. Tuberculosis
223. 62-year-old female has kidney stone and treated with PCNL.
c. Schistosomiasis d. Trauma
After 2 days she again comes to OPD with chills and fever.
What is the complication? 231. The following are the complications of PCNL except
a. Bacterial sepsis b. Acute pancreatitis a. Colon Injury b. Pleural injury
c. Splenic injury d. Ureteric stricture c. Bleeding d. Ureteric Stricture
XXXVIII
Latest Questions Paper 2020-2019
UPSC JUNE 2019 240. What is true about the management of a corrosive injury of
oesophagus?
232. What is the correct order of the normal phases of wound a. Early-skilled endoscopy is must
healing? b. Immediate surgery with oesophagectomy is advisable
a. Proliferative phase → Haemostatic phase → Inflammatory c. Broad spectrum antibiotics should be started as soon as
phase → Remodelling phase possible
b. Hemostatic phase → Inflammatory phase → Proliferative d. Immediate NG tube insertion and gastric lavage should be
phase → Remodelling phase performed
c. Destructive phase → Proliferative phase → Remodelling 241. Which of the statements regarding Salivary gland neoplasms
phase → Inflammatory phase are correct?
d. Remodelling phase → Proliferative phase → Destructive 1. 80–90% of parotid tumors are malignant
phase → Inflammatory phase 2. 90 % of sublingual gland tumors are malignant
233. All of the following are risk factors for an increased risk of 3. 60–70 % of submandibular gland tumors are benign
wound infection EXCEPT: 4. Parotid gland is most common site for salivary gland
a. Obesity b. Hypertension tumors
c. Jaundice d. Cancer Select the correct answer using the code given below:
a. 1, 2 and 3 b. 2, 3 and 4
234. Systemic Inflammatory Response Syndrome (SIRS) is c. 1, 3 and 4 d. 1, 2 and 4
characterized by all of the following EXCEPT:
a. Hyperthermia (>38°C) 242. A few days following viral fever, a 50-year-old female
b. Platelet count (<1,00,000/mm3) presented with pain in neck, fever, malaise and firm
c. Tachypnoea (>20/min) enlargement of both the lobes of thyroid. On investigation
d. Hypothermia (<36°C) thyroid antibodies were normal & serum T4 was high
normal. Probable diagnosis is:
235. Gentleman of 56 years underwent laparoscopic left a. Autoimmune thyroiditis
hemicolectomy for diagnosed left colonic carcinoma. b. Lymphoma of thyroid
Histopathology revealed the tumour to be invading c. Granulomatous thyroiditis
submucosa and muscularis propria. Among the 16 regional d. Riedel’s thyroiditis
lymph nodes harvested, 2 were positive for malignant
deposits. His staging as per AJCC will be: 243. No increased relative risk of invasive breast carcinoma based
a. T1, N1, M0 b. T2, N1, M0 on histopathological examination of benign breast tissue is
c. T1, N0, M0 d. T2, N1, M1 for all of the following EXCEPT:
a. Hyperplasia
236. All of the following are hormonal agents used in treatment b. Periductal mastitis
of cancer EXCEPT: c. Squamous metaplasia
a. Anastrazole b. Irinotecan d. Solitary papilloma of lactiferous sinus
c. Cabergoline d. Leuprolide
244. Which of the following statements regarding Paget’s disease
237. Which of the following statements regarding lymphoedema of nipple are correct?
are correct? 1. It represents benign pathology of nipple areola complex
1. Patients experience constant dull ache and even severe 2. It is eczema like condition of nipple and areola
pain sometimes 3. Erosion of nipple is seen
2. Manual lymphatic drainage has a role 4. Nipple biopsy is required for definitive diagnosis
3. Primary lymphoedema is caused by congenital lymphatic
dysplasia Select the correct answer using the code given below:
4. Nonne-Milroy’s disease is a type of primary lymphoedema a. 1, 2 and 3
b. 2, 3 and 4
Select the correct answer using the code given below: c. 1, 3 and 4
a. 3 and 4 only b. 1 and 2 only d. 2 and 4 only
c. 1, 2 and 3 only d. 1, 2, 3 and 4 245. A gentleman of 36 years presented with a long history
238. Indications for carotid endarterectomy in symptomatic of upper abdominal pain which was periodic and often
patients are all of the following EXCEPT: occurred early morning. For last 3 months, he is having
a. Hemianopia b. Monocular blindness projectile vomiting, which is non bilious, unpleasant in
c. Dysphasia d. Persistent hypertension nature with undigested food materials. On examination he
appears unwell, dehydrated and seemed to have lost weight.
239. A policeman of 45 years presented with Lipodermatosclerosis
Probably he is suffering from:
over lower medial aspect of left leg, along with a healed
a. Gastric outlet obstruction
venous ulcer. As per the CEAP (Clinical-etiology-anatomy
b. Carcinoma stomach
pathophysiology) classification his clinical classification
c. Gastro-oesophageal reflux with oesophagitis
will be:
d. Superior mesenteric artery syndrome
a. C4a b. C4b
c. C5 d. C6
XXXIX
Surgery Sixer for NBE
246. A 40-year-old female patient presents with colicky 251. A young sports person presented in surgical emergency
abdominal pain associated with episodes of mild diarrhoea with the complaints of severe pain in the groin area,
for last 6 months accompanied with intermittent fever extending into the scrotum and upper thigh. The pain is
and weight loss. There are multiple discharging sinuses on debilitating and he is notable to exercise. On examination
perineal examination. The most likely clinical diagnosis in there is tenderness in the region of Inguinal canal and pubic
this patient is: tubercle. He is probably suffering from:
a. Amoebic colitis b. Crohn’s disease a. Varicocele
c. Ulcerative colitis d. Ileocaecal tuberculosis b. Inguinal hernia
247. A gentleman of 48 years was being worked up for c. Sportsman hernia
hepatocellular function. He had no history or signs of d. Femoral hernia
encephalopathy. His serum bilirubin was 5 mg%, serum 252. Ventral hernia includes all EXCEPT:
albumin was 3.9 gm%, International normalized ratio a. Epigastric hernia
was 1.6. On ultrasound no free fluid was detected inside b. Para-umbilical hernia
abdomen. As per Child-Turcotte-Pugh (CTP) classification, c. Lumbar hernia
he was in: d. Inguinal hernia
a. CTP–A b. CTP–B 253. Diaphragmatic injury is suspected in a 50-year-old
c. CTP–C d. CTP–D gentleman with history of blunt abdominal trauma, having
248. Which of the statements regarding Calot’s triangle are a normal chest X-ray. He is best managed by:
correct? a. Diagnostic peritoneal lavage and proceed
1. Common hepatic duct forms the medial boundary of the b. Upper GI contrast study
Calot’s triangle c. CECT abdomen
2. Inferior surface of the right lobe of the liver forms the d. Diagnostic laparoscopy
superior boundary of Calot’s triangle 254. A 20-years-old man had history of pain in the right side of
3. Right hepatic artery is usually found as a content of the abdomen. His X-ray abdomen AP view shows radio-opaque
Calot’s triangle shadow, which on lateral film falls behind the vertebral
4. Cystic duct and medial border of gallbladder forms the column. The probable diagnosis is:
lateral border of Calot’s triangle a. Gall stone disease
Select the correct answer using the code given below: b. Renal calculus
a. 1, 2 and 3 b. 2, 3 and 4 c. Calcified mesenteric lymph node
c. 1, 3 and 4 d. 1, 2 and 4 d. Phlebolith
249. Consider the following statements regarding Opportunistic 255. Urinary bladder can be injured in all of the following
post-splenectomy infections (OPSI): operations EXCEPT:
1. Haemophilus influenzae, Neisseria meningitidis and a. Inguinal hernia repair
Streptococcus pneumoniae are the most common b. Hysterectomy
causative agents c. Surgery for rectum
2. Risk is greatest in the patients who have undergone d. Inguinal lymph node dissection
splenectomy for trauma 256. In diffuse axonal injury (DAI) all are true EXCEPT:
3. Risk is greatest within the first 2–3 years following a. Form of primary brain injury
splenectomy b. Seen in high energy
4. Prophylactic vaccination should be done 2 weeks prior to c. Patient is comatose
elective splenectomy d. CT scan shows pathognomonic finding
Which of the statements given above are correct? 257. A 70 years old man on anticoagulants due to some heart
a. 1, 2 and 3 b. 2, 3 and 4 disease suffered a minor head injury. One month later he
c. 1, 3 and 4 d. 1, 2 and 4 has severe headache with slowly developing neurological
250. A 48-year-old male with the history of chronic duodenal signs. The probable diagnosis is:
ulcer presented in surgical emergency with the complaints a. Extradural hematoma
of sudden severe pain in the abdomen. At presentation: b. Acute subdural hematoma
Pulse = 120/m, BP = 90/60 mm of Hg. Abdomen: Tenderness c. Chronic subdural hematoma
(+), Rigidity (+), Guarding (+)Respiratory Rate: 20/m X-ray: d. Subarachnoid hemorrhage
Gas under right dome of diaphragm 258. In Split thickness graft, which part of the skin is/are
The probable diagnosis is: included?
a. Acute appendicitis a. Epidermis only
b. Acute pancreatitis b. Epidermis and dermis
c. Acute myocardial infarction c. Epidermis and part of dermis
d. Perforation peritonitis d. Epidermis, dermis and part of sub cutaneous tissue
XL
Latest Questions Paper 2020-2019
259. A 20-year-old patient underwent open hernia surgery 261. Refeeding syndrome seen after enteral or parenteral
four days back. He is running fever for the last one day nutrition is characterized by all EXCEPT:
and on local examination the operated site was wet with a. Hypophosphatemia
pus and surrounding redness and edema. The appropriate b. Hypocalcemia
management would be: c. Hypomagnesemia
a. Change of antibiotics d. Hyponatremia
b. Daily dressing 262. The capillary refill time is prolonged in all types of shock
c. Opening sutures and cleaning of wound EXCEPT:
d. Sending pus for C/S a. Hypovolaemic shock
260. Good surgical practice and surgical ethics include all b. Cardiogenic shock
EXCEPT: c. Septic shock
a. Respect autonomy b. Informed consent d. Obstructive shock
c. Confidentiality d. Experiment
T
he important topics have been marked with *, ** and ***
Tension pneumothorax is the diagnosis from the History: •• CECT is a part of secondary survey
•• Bailey and Love says- Needle Insertion in 2nd ICS in Mid •• For major Trauma we must advise WBCT (Whole Body
Clavicular line CT) if they are stable.
•• ATLS- 10th Edition says – 5th ICS in Mid axillary line in
adults and 2nd ICS in Mid clavicular line in Children. 5. Ans. (a) Battle sign
3. Ans. (b) CECT abdomen Ref: Surgery Sixer 4th Edition Page 88
Ref: Surgery Sixer 4th Edition Page 95 •• Fracture of Middle cranial fossa shows mastoid bruising
Penetrating trauma Protocol is shown in the following known as battle Sign**
XLI
flowchart.
Surgery Sixer for NBE
Section B: Head and Neck, Thyroid and Breast This fascia limits the extension of thyroid swelling above.
This fascia provides fascial sheath for thyroid, Oesophagus,
6. Ans. (c) Inferior Belly of Omohyoid trachea and go down and fuse with superior mediastinum.
Ref: Surgery Sixer 4th Edition Page 187 This Prevents Gland enlarging superiorly but allows to
enlarge Retrosternally.
•• The first step in Superficial Parotidectomy is to identify the •• Berry ligament is attached to Cricoid Cartilage- is a
facial nerve Trunk: condensation of Pre tracheal fascia and binds thyroid to
•• The following methods are used to identify the Facial Nerve: Trachea. This is the point where the risk of injury to RLN
Antegrade methods: is highest. RLN enters the Larynx at level of Cricothyroid
•• Inferior Portion of Tragal Pointer (CONLEY’S POINTER) – Joint. This ligament is one of the important reasons for
Facial nerve lies 1cm Deep and inferior to its tip gland to move with deglutition.
•• Superior to point of insertion of Posterior Belly of Digastric
Muscle.
•• Squamotympanic Fissure
•• Styloid Process (Facial Nerve found Superficial to this)
Bailey's 27/e update
•• Mastoid process can be drilled and nerve identified more Investigations for Lung Cancer:
proximally. Non Invasive investigations:
Retrograde method: Chest X-ray:
•• It will detect mostl cancers
•• To identify the Buccal Branch or Cervical Branch of Facial
•• Secondary effects like pleural effusion, distal collapse and
nerve and trace backwards.
raised hemidiaphragm.
•• Very useful in re-operative cases.
CT scan thorax:
7. Ans. (b) All should undergo CT chest •• 1st investigation of suspected lung cancer
•• Look for resectability
Ref: Bailey and love 27th edition Page 811 •• Lymphnodes >2 cm in diameter are likely to be metastatic.
PET- CT
Sputum Cytology
Bailey's 27/e update Invasive investigations:
Retrosternal Goitre Bronchoscopy:
•• MC cause is slow growth of MNG downwards into •• Transbronchial needle aspiration can be done to confirm the
mediastinum. tumor
•• It can lead to compression of Trachea, esophagus and can Endobronchial USG:
cause dyspnea and dysphagia respectively. •• Suspicious mediastinal nodes are biopsied.
•• In elderly patients with incidentally discovered retrosternal CT guided biopsy:
goitre can be observed without surgery. •• Percutaneous CT guided biopsy is ideal for peripheral and
•• >95% patients are operated by transcervical incision only. larger tumors to study the histology
•• Median sternotomy is done for selected patients like •• Pneumothorax is a complication.
Suspected malignancy, Revisional surgery, or those extending Surgical Diagnosis and Staging:
into posterior mediastinum/ Fixed and immobile.
Mediastinoscopy:
•• All cases must undergo Cross sectional imaging with CECT •• Under GA, transverse incision made above sternal notch,
Chest** careful dissection in this plane will show paratracheal and
•• They all derive blood vessels – from Thyroid arteries. subcarinal nodes
•• Mediastinoscope is inserted and biopsy done
•• Complications—Pneumothorax and hemorrhage
8. Ans. (b) Hashimoto’s thyroiditis
Mediastinotomy:
Ref: Surgery Sixer 4th Edition page 226 •• 2nd IC space an incision is made to access mediastinal nodes
•• MC cause of Hypothyroidism in Western countries- •• On left paraaortic nodes and subaortic fossa entered.
Hashimoto’s Thyroiditis VATS approach:
•• MC cause of Hypothyroidism in Iodine Deficient areas- •• Staging the mediastinum and take node sampling
Iodine Deficient Colloid Goitre •• Assess the resectability of cancer lung.
•• Hashimoto’s on FNAC shows Hurthle cells and Lymphocytic BEAHR’S Triangle:
infiltration in the gland. •• Tracheo Oesophageal Groove
•• Antibodies like Thyroid peroxidase (TPO- Most •• Carotid Artery
Commonly), TSH- R are elevated. •• Inferior Thyroid Artery
Importance of this triangle- This is the surgical anatomy to
9. Ans. (d) Pretracheal fascia
find our Recurrent Laryngeal nerve.
Ref: B.D. Chaurasia Another place where nerve can be identified is at most
•• Pre tracheal fascia extends from base of skull posteriorly posterolateral portion of gland- Tubercle of Zuckerkandl**
XLII and hyoid bone and thyroid cartilage anteriorly and laterally.
Latest Questions Paper 2020-2019
10. Ans. (a) Papillary cancer Type 2 Achalasia •• Elevated median IRP (>15 mm Hg),
Ref: Surgery Sixer 4 Edition Page 226
th with Esophageal •• 100% failed peristalsis,
Compression •• Pan oesophageal pressurisation with
FNAC and Thyroid gland: ≥20% of swallows
•• Hurthle cells- Hashimoto’s thyroiditis Contractions may be masked by
•• Orphan Annie Eyed Nuclei or Psammoma Body- Papillary oesophageal pressurisation and DCI should
cancer not be calculated**
•• Bethesda Classification Thy 3- Follicular Neoplasm Type 3 (Spastic •• Elevated median IRP (>15 mm Hg),
•• Amyloid Stroma- Medullary cancer Achalasia) •• No normal peristalsis,
•• Impossible to do FNAC- Anaplastic cancer •• Premature (spastic) contractions with DCI
•• Trucut Biopsy advised- Anaplastic cancer >450 mm Hg cm with ≥20% of swallows
May be mixed with pan oesophageal
11. Ans. (b) DIEP flap pressurisation**
Ref: Surgery Sixer 4th Edition page 272
EGJ Outflow •• Elevated median IRP (>15 mm Hg),
Autogenous Transplant for Breast Reconstruction obstruction •• Sufficient evidence is seen for peristalsis
(i) Abdominal based flap: such that criteria for types I–III achalasia
Transverse rectus abdominis myocutaneous flap: TRAM are not met
flap •• Question paper had- DCI >450 mm Hg cm, Pan
Free flap (based on inferior epigastric A)
Esophageal pressurisation also and hence go with Type 3
Pedicled flap (based on superficial epigastric A)
Achalasia cardia.
In these flaps we cut the rectus muscle and hence weakness •• No more controversy please
of abdominal wall happens. To avoid this a new innovative
flap known as 14. Ans. (c) Spontaneous rupture of esophagus
“DIEP Flap- Deep inferior Epigastric Artery Perforator
Ref Surgery Sixer 4th Edition Page 318
Free Flap, in which we remove only the skin and
Subcutaneous tissue in the lower abdomen and the muscle The triads seen in Boerhaave syndrome:
is not removed”- This DIEP flap is the best as on date now. Mackler’s Triad:
(ii) Latissimus dorsi flap (based on thoracodorsal artery) •• Sudden vomiting
(iii) Gluteal flap (RUBEN’S FLAP based on Circumflex Iliac •• Chest pain
artery) •• Subcutaneous emphsyema
12. Ans. (c) Colloid Anderson Triad:
XLIII
Surgery Sixer for NBE
XLIV
Latest Questions Paper 2020-2019
•• Identify the prime cause and remove it. •• MC functioning endocrine neoplasm of pancreas-
•• Fluid and electrolyte correction Insulinoma
•• If no dynamic cause is seen we can use CATCHPOLE •• MC endocrine neoplasm in MEN -1 Syndrome- Gastrinoma
Regimen**- Stimulating intestine movement with •• MC malignant endocrine neoplasm- gastrinoma
Adrenergic Blocking agent**+ Cholinergic Stimulation •• MC neuroendocrine tumor- Non-functioning tumors
Using Neostigmine.
24. Ans. (c) Duodenum
21. Ans. (c) Failure of Neural crest cells to migrate Ref: Surgery Sixer 4th Edition Page 968
Ref: Surgery Sixer 4 Edition Page 426
th
MEN-1 Syndrome (Werner Syndrome)
Hirschsprung Disease: •• Autosomal Dominant syndrome
•• Incidence 1 in 5000 •• 2/ 1 lakh population Prevalence.
•• Male: Female = 4:1 ratio •• 80% cases have Truncated menin protein** and result in
•• The main etiology is failure of migration of neural crest. oncogenesis
•• Commonly associated with RET Oncogene mutation. Clinical Features:
•• Associated with Down Syndrome and MEN 2 syndromes.
•• Affects 3 P- Parathyroid (90%-MC), Pituitary and Pancreas.
•• Pathology- Absence of Ganglion cells in both myenteric
•• 90% have Hyperparathyroidism due to Multiglandular
and submucous plexus with accompanying hypertrophy of
hyperplasia*
nerve trunks and increased acetyl cholinesterase**
•• 2nd MC manifest is Neuroendocrine tumor of Pancreas and
•• Associated with DOWN syndrome**
Duodenum. (30-80%)
•• Carcinoids in Thymus and Bronchus
Section D: Hepatobiliary System •• Pituitary adenomas- MC prolactinoma (15%-30%)
•• Male and Female equally affected.
•• MC pancreatic tumor is Gastrinoma**
22. Ans. (a) ERCP
•• MC cause of mortality in MEN-1 is NET from Duodenal
Ref: Surgery Sixer 4th Edition page 548 and pancreatic tumors or Intra thoracic carcinoids.
•• The Best method to manage the CBD stone is ERCP, •• MEN 1 manifests from age of 5 years*
Sphincterotomy and Stone removal. •• Biochemical surveillance to start at below ages:
•• The question based on the Protocol for CBD stone Shown 5 years: Prolactin level
below. 8 years: PTH and Calcium Level
20 years: Gastrin, Chromogranin, Glucagon and Proin-
sulin
Hyperparathyroidism:
•• MC manifest in MEN -1 (Fig. 1)
•• Due to multiglandular hyperplasia**
•• MC in 2nd decade*
•• 1st Biochemical abnormality noted is Hypercalcemia*
•• Enlargement is asymmetrical** (Figure)
•• Treatment: 2 options:
Total Parathyroidectomy with heterotopic implantation
of one parathyroid in Brachioradialis muscle
Subtotal (removal of 3 and half glands)- Advantages are
single incision and absence of transient Hypocalcemia
23. Ans. (a) Insulinoma which may be seen in Total Parathyroidectomy**
Ref: Surgery Sixer 4th Edition Page 443
XLV
Surgery Sixer for NBE
•• MC cause is Atheromatous degeneration The face of mickey mouse is Femoral vein and the medial ear
•• MC symptom – Back and Abdominal Discomfort** is saphenous vein and lateral ear is Femoral artery as shown
•• Site of rupture: M/c site = posterolateral (80%)* below.
•• 2nd m/c site = anterior (20%)
•• Less than 50% of patients with rupture survive to reach
hospital.
•• Anterior rupture results in free bleeding into the peritoneal
cavity; very few patients reach hospital alive.
•• Posterior rupture on the other hand produces a
retroperitoneal hematoma**
•• Ruptured abdominal aortic aneurysm is a surgical
emergency; it should be suspected in a patient with the triad
of severe abdominal and/or back pain, hypotension and a
pulsatile abdominal mass**
•• Bailey and Love says “It should always be remembered
that the treatment of ruptured aneurysm is operation, not
monitoring and resuscitation”.
Indications for surgery
•• All symptomatic aneurysm (m/c symptom is back and
abdominal discomfort)
•• Aneurysms size greater than 55 mm* on USG antero-
posteriorly.
•• Aneurysms <55 mm in diameter needs Regular USG 33. Ans. (b) Chronic Lymphedema
assessment.
Ref: Surgery Sixer 4th Edition page 842
30. Ans. (a) Aortoiliac obstruction Clinical Features of Lymphedema:
Ref: Bailey and Love 27th Edition Page 944 Table •• Lymphedema Characteristically involves foot- Buffalo
•• Aortoiliac obstruction causes Claudication pain in both/ Hump forms in foot
BILATERAL Calf, Thigh and Buttocks** •• Toes appear Square.
•• Iliac Obstruction causes Claudication pain unilateral calf •• Stemmers sign*- inability to pinch subcutaneous tissue in
and thigh with occasional Buttocks. lymphedema
•• Buttock Claudication + Impotence seen in Saddle •• Initially there is pitting seen but later Fibrosis and
Embolus at bifurcation- the syndrome is known as Leriche Hyperkeratosis happens and pitting becomes absent.
syndrome** •• Fungal infection and Onychomycosis (nail Fungi) are seen.
•• Lymphangiomas- Blisters of dilated Dermal Lymphatics on
31. Ans. (b) Most common site is calf the skin, Fluid is usually clear.
Ref: Bailey and Love 27th Edition Page 943 •• If lymphangiomas <5 cm it is named as Lymphangioma
Circumscriptum**, Widespread one known as Lymphan-
Intermittent claudication: gioma Diffusum**
•• Occurs due to anaerobic muscle metabolism, described as •• Reticular pattern of ridges is known as Lymphedema ab
cramp like pain in muscles. igne**
•• Brought by walking Brunner Grading of Lymphedema:
•• Not present on 1st step
•• Relieved by rest both in sitting or standing positions within •• Grade 0- Histological abnormalities only, No Clinical
5 minutes evidence
•• Claudication distance may vary only slightly from day to •• Grade 1- Pitting Edema, Subsides on Elevation
day. •• Grade 2- Non-Pitting edema, Not relieved with elevation
•• Claudication distance is decreased by walking up hills, •• Grade 3- Irreversible skin changes, fibrosis, papillae
increasing speed of walking, carrying heavy weights,
34. Ans. (d) High Risk
anemia, heart failures.
•• MC affected part is calf* (as 70% cases superficial femoral Ref: Surgery Sixer 4th Edition page
artery is obstructed) Please remember there are two Modified Well’s criteria in
MC cause for Claudication is Superficial Femoral Artery Bailey and Love- one for DVT risk prediction and another for
Occlusion** and hence mc site is Calf** finding the probability of Pulmonary embolism in a known
case of DVT.
32. Ans. (c) Mickey mouse sign Modified Well’s Criteria for predicting Deep vein
Ref: Surgery Sixer 4th Edition Page 846 thrombosis:
XLVII
Surgery Sixer for NBE
Urology
XLVIII
Latest Questions Paper 2020-2019
XLIX
Surgery Sixer for NBE
46. Ans. (a) Subarachnoid hemorrhage •• The patient may also be hypothermic.
•• Warming should be gentle as the heat used may actually
Ref: Surgery Sixer 4th Edition page 903
cause a burn!
The History is clear-It’s due to rupture of berry aneurysm
Management
– there is subarachnoid hemorrhage happening which is
characterised by Sudden Head ache and Neck Stiffness. The •• Frostbite injuries affect the peripheries in cold climates.
CT Scan image shows the Hemorrhage in Subarachnoid space. •• The initial treatment is with rapid rewarming in a bath
at 42°C**
Order of Frequency
•• Warming takes 15 minutes to 1 hour. Rewarming can be
•• Anterior communicating artery: Anterior cerebral junction— very painful, so pain management is important.
most common site •• Rehydrate with warm fluids
•• Posterior communicating artery: Internal carotid junction •• NSAIDs
•• Middle cerebral bifurcation •• Demarcation between Dead and Viable tissue will occur.
•• Intracranial carotid bifurcation •• Surgery is not done at this phase. We will wait for recovery.
•• Vertebrobasilar or basilar bifurcation •• Keep injured area—Clean and dry and prevent further
Multiple in 20–30% cases injury or infection.
Predisposing factors: Smoking and hypertension •• Recent advances—Tissue plasminogen activator and nerve
blocks help in reducing amputations but should be started
47. Ans. (a) Superficial spreading type in 24 hours.
Ref: Bailey and Love 27th Edition Page 610 Surgical Management
•• The history is clear—the malignant melanoma arises •• Definitive surgery to excise dead tissue can be left for
from a long standing naevi, and this line is direct line many months.
from Bailey showing Superficial spreading type.
•• The following are the features in naevi suggestive of 49. Ans. (c) Colonic Diverticulitis
malignant melanoma:
Ref: Multiple Sources
Change in size
Change in shape •• Tolvaptan:
Change in color Vasopressin Receptor antagonist
Thickness (Elevated, nodularity or ulceration) 1st Drug approved for use in ADPKD
Satellite lesions, spreading of pigment Increases amount of urine excretion.
Tingling, itching and discharge (Late signs**) Increases body sodium level and hence can be useful
•• Most common type of Malignant melanoma is Superficial in Hyponatremia like in heart failure, Liver disease and
spreading type, mostly arises from long standing pre- certain hormonal imbalances.
existing naevi (mole). There is no dangerous side effect mentioned on treatment
•• If nodularity is seen it represents vertical growth. like that of abdominal pain.
Hence the first choice is ruled out**
48. Ans. (b) 42°
Ref: Bailey and Love 27th Edition Page 423, 632
Please understand 2 diseases- Frost bite and Trench foot: Bailey's 27/e update
Frost bite Autosomal Dominant Polycystic Kidney Disease (ADPKD)
•• Multisystem disorder
•• Occurs due to body freezing
•• Almost Bilateral in 95% cases.
•• Cells are disrupted and tissues die
•• Associated with cysts in Liver, Pancreas, Arachnoid
•• It is a type of COLD BURN and can be categorised based on
membranes.
depth like a normal burns.
•• It is a genetic condition caused by mutations of PKD1 and
•• Other mechanisms which play the role are- Vasoconstriction,
PKD2 genes.
Capillary sludging and reperfusion injury at rewarming
•• Autosomal dominant type.
phase.
•• Prevalence: 1:400 to 1:1000
•• MC site involved is Fingers**, Toes, Cheeks, Nose and ears.
•• Disease does not manifest below the age of 30 years.
•• Frozen tissue is hard and indentation not possible.
Immersion injury (Trench foot)
Diagnosis:
•• Immersion injury is a cold injury, which does not involve
•• Renal USG shows—at least 3 unilateral or Bilateral renal
actual freezing of the tissue and is commonly caused by
cysts and 2 cysts in each kidney are sufficient for diagnosis.
prolonged immersion in cold water (hence trench foot).
L
Latest Questions Paper 2020-2019
Clinical Features:
LI
Surgery Sixer for NBE
52. Ans. (a) Southampton scoring system Type of Examples of Infection Infection
Ref: Surgery Sixer 4th Edition Page 38 Surgery Surgeries rate with rate without
•• There are two scoring system in Surgical Site infection Prophylaxis prophylaxis
categorisation: Clean Surgery: •• Heart, 1-2% 1-2%
Southampton Grading (No viscus Brain, Joint,
ASEPSIS Score opened) Transplant
Southampton Wound grading System: surgeries
•• Herniorrhaphy*
Grade Appearance •• Swelling
0 Normal healing excision
LII
Latest Questions Paper 2020-2019
•• Mask or respirator
Front of mask/respirator is contaminated
DO NOT TOUCH
Grasp bottom, then top ties or elastics and
remove
Discard in waste container
Also remember wearing order now itself for next year exam:
•• Gown- Mask- Face shield- Gloves
•• Gown
Fully cover torso from neck to knees, arms to
end of wrists, and wrap around the back
Fasten in back of neck and waist
Contd...
LIII
Surgery Sixer for NBE
•• Mask or respirator
Secure ties or elastic bands at middle of head
and neck
Fit flexible band to nose bridge
Fit snug to face and below chin
Fit-check respirator
•• Gloves
Extend to cover wrist of isolation gown
58. Ans. (a) Jackson-Pratt Drain (New Question- Internet 61. Ans. (d) 1-3-2-4
Sources referred) Ref: Internet Sources
Jackson-Pratt Drain: This is WHO Protocol for Handwashing as per this protocol
•• A Jackson-Pratt drain (or JP drain) is rubber tubing that may •• Palms
be placed after surgery. It may also be used with infections •• Dorsum of hand
or injury that can cause a build-up of fluid. •• Fingers
•• Fluid that collects inside the body can increase the chance of •• Finger nails
infection or other complications. The JP drain allows fluids •• Thumb
to move out of the body.
LIV
Latest Questions Paper 2020-2019
LV
Surgery Sixer for NBE
LVI
Latest Questions Paper 2020-2019
80. Ans. (a) False (b) False (c) True (d) True
Ref: Surgery Sixer 4th Edition Page 740
In terms of Lobes in Prostate- there are 5 lobes:
•• Prostatic cancer arises from Posterior lobe**
•• BPH arises from Medial Lobe.
In terms of Zones in prostate – there are 3 zones:
•• Carcinoma arises from Peripheral zone**
•• BPH arises from Transitional Zone
Gleason Score for Prostatic Cancer:
•• MC histological Type of Prostatic cancer is Adenocancer.
•• Classification of histological pattern based on the degree of
glandular de- differentiation and its relation to stroma has
been devised by GLEASON.
•• Prostate cancers exhibit heterogeneity within the tissue
itself and hence two histological areas of prostate are scored
each between 1-5.
•• The scores are added to an overall score of 2-10.
•• Gleason score 2-10 helps to predict the likelihood of spread
and prognosis
•• Platelets are not of any use in a case of bleeding esophageal •• Example: Gleason Score of 8-10 : High Grade tumor**, Poor
varices* Prognosis**
LVII
Surgery Sixer for NBE
82. Ans. (b) Femoral artery JIPMER DECEMBER AND MAY 2019
•• MC site of Pseudoaneurysm is femoral artery due to blood
sampling and improper pressure in femoral artery by
residents. Section A: General
83. Ans. (c) Acute respiratory failure 88. Ans. (d) Stage 2 and 3 ulcers may be left to heal secondarily
•• One of the well-known complications in post-operative after debridement
period is acute respiratory failure after CABG operation
Ref: Schwartz 10th Edition Page 482
84. Ans. (b) Skin grafting
Pressure Sore
•• This is a well-granulated venous ulcer. Partial thickness
graft with splitting is ideal for this type wound. Tissue pressure >30 mm Hg will result in Tissue Ischemia.
Most Common Site is Ischial tuberosity (28%), 2nd Most
85. Ans. (a) 1-2-4-3 common site is Greater Trochanter (19%) and 3rd Common site
Ref: Bailey and Love 27th edition Page 1414 is Sacrum. Muscle is more susceptible than skin to ischemic
insult due to its relatively high metabolic demand.
This question from direct Bailey and Love- We will follow the
order in Bailey and Love as in list shown below. Stages of Pressure Sore
Grades of Renal Laceration: •• Stage 1, Non-blanching erythema over intact skin;
•• Stage 2, Partial-thickness injury (epidermis or dermis)—
•• Grade 1- Subcapsular Non Expanding Perirenal hematoma
blister or crater;
•• Grade 2- Laceration <1 cm, Non-Expanding Perirenal
•• Stage 3, Full-thickness injury extending down to, but not
hematoma confined to retroperitoneum and Gerota fascia
including, fascia and without undermining of adjacent
•• Grade 3- Laceration >1 cm, No urine extravasation
tissue; and
•• Grade 4- Laceration with Urine Extravasation, Injury to
•• Stage 4, Full-thickness skin injury with destruction or
Main renal artery and Vein
necrosis of muscle, bone, tendon, or joint capsule.
Grade V:
Management
•• Shattered kidney.
•• Management principles for pressure sores should
•• Avulsion of renal hilum: Devascularisation of a kidney due
include pressure relief (air mattresses and gel cushions
to hilar injury.
for redistribution of pressure), systemic optimization
•• Ureteropelvic avulsions.
(particularly nutritional support), and wound care.
•• Complete laceration or thrombus of the main renal
•• Goals of surgical intervention are drainage of fluid
artery or vein.
collections, wide debridement of devitalized and scarred
tissue, excision of pseudo-bursa, ostectomy of involved
86. Ans. (a) Melanoma
bones, hemostasis, and tension-free closure of dead space
Ref: Surgery Sixer Page 929 with well-vascularized tissue (muscle, musculocutaneous,
•• Asymmetry or fasciocutaneous flaps).
•• Border irregularity •• Stage 2 and 3 ulcers may be left to heal secondarily after
•• Color variation debridement**.
•• Diameter increased >6 mm •• Sub-atmospheric pressure wound therapy devices (vacuum-
•• Evolution assisted closure) play a role in wound management by
removing excess interstitial fluid, promoting capillary
87. Ans. (d) Explore both sides immediately circulation, decreasing bacterial colonization, increasing
Ref: Surgery Sixer page 752 vascularity and granulation tissue formation, and
•• Both affected and unaffected side must be explored contributing to wound size reduction.
simultaneously and repaired immediately.
LVIII
Latest Questions Paper 2020-2019
LIX
Surgery Sixer for NBE
Berkow Formula to Estimate Burn Size (%) Based on Area of Burn in an Isolated Body Part
Body Part 0-1 Years 1-4 Years 5-9 Years 10-14 Years 15-18 Years Adult
Head 19 17 13 11 9 7
Neck 2 2 2 2 2 2
Anterior trunk 13 13 13 13 13 13
Posterior trunk 13 13 13 13 13 13
Right buttock 2.5 2.5 2.5 2.5 2.5 2.5
Left buttock 2.5 2.5 2.5 2.5 2.5 2.5
Genitalia 1 1 1 1 1 1
Right upper arm 4 4 4 4 4 4
Left upper arm 4 4 4 4 4 4
Right lower arm 3 3 3 3 3 3
Left lower arm 3 3 3 3 3 3
Right hand 2.5 2.5 2.5 2.5 2.5 2.5
Left hand 2.5 2.5 2.5 2.5 2.5 2.5
Right thigh 5.5 6.5 8 8.5 9 9.5
Left thigh 5.5 6.5 8 8.5 9 9.5
Right leg 5 5 5.5 6 6.5 7
Left leg 5 5 5.5 6 6.5 7
Right foot 3.5 3.5 3.5 3.5 3.5 3.5
Left foot 3.5 3.5 3.5 3.5 3.5 3.5
LX
Latest Questions Paper 2020-2019
95. Ans. (a) All trauma Patients 97. Ans. (b) Stage II
(Surgery Sixer page 124, Repeat JIPMER Question) Ref: AJCC 8th Edition page 883
•• Patient’s with Trauma – Airway management is first done AJCC 8th Edition: Thyroid Cancer Staging:
alone with Cervical spine stabilisation.
T staging
•• All trauma patients – unless proved otherwise consider
•• Tx- Primary tumor cannot be assessed
them as vertebral column is injured, and take appropriate
•• T0- No evidence of primary tumor
precautions. Carefully apply a hard-cervical collar and use
T1- Tumor <2 cm in greatest dimension limited to the
LOG ROLL (rotating the patient as a whole while examining
thyroid
the spine).
•• T1a - Tumor <1 cm
•• Rigid long boards with straps are helpful in transporting
•• T1b - Tumor 1 cm-2 cm
the patient from one place to other. Conscious and alert
T2- Tumor 2-4 cm limited to the thyroid
patient indirectly means they don’t need Intubation and
their airway is patent. T3- Tumor >4 cm or with extrathyroidal extension
•• T3a: Tumor >4 cm in greatest dimension limited to the
thyroid
Section B: Head and Neck, Thyroid, Breast •• T3b: Tumor of any size with gross extrathyroidal extension
invading only strap muscles (sternohyoid, sternothyroid,
96. Ans. (a) Spinal Accessory Nerve thyrohyoid or omohyoid muscles)
T4: Locally Advanced
Ref: Bailey and Love 27th Edition Page 758
•• T4a: Moderately advanced disease; tumor of any size with
Radical Neck Node Dissection of CRILE’S: gross extrathyroidal extension into the nearby tissues of the
The classic operation involves resection of the cervical neck, including subcutaneous soft tissue, larynx, trachea,
lymphatics and lymph nodes and those structures closely esophagus, or recurrent laryngeal nerve.
associated: •• T4b: Very advanced disease; tumor of any size with
•• The internal jugular vein, extension toward the spine or into nearby large blood
•• The accessory nerve,
vessels, invading the prevertebral fascia, or encasing the
•• The sternocleidomastoid muscle.
carotid artery or mediastinal vessels.
These structures are all removed en bloc and in continuity with
All T categories must be subdivided into Solitary(S) or
the primary disease, if possible.
Multifocal(M), in Multifocal- Largest dimension taken for
The main disability that follows the operation is weakness
T staging.
and drooping of the shoulder due to paralysis of the trapezius
muscle as a consequence of excision of the accessory nerve.
LXI
Surgery Sixer for NBE
LXII
Latest Questions Paper 2020-2019
•• I123 has half-life of 12-14 hours* and emits low dose radiation Ref: Surgery Sixer 4th Edition Page 411.
hence used to detect Ectopic thyroids and nodular goiter Malignant carcinoid syndrome develops only liver
activities. metastases develop; because all the hormones are detoxified
•• I131 has long half-life of 8 days hence used to diagnose and in liver. The only exceptions to get Carcinoid syndrome in
treat differentiated thyroid cancers. absence of Liver mets are at – Ovarian and Retroperitoneal
•• Diagnostic dose- <5 micro curie, therapeutic doses may be carcinoids as their drainage is directly systemic and not
given up to 200 micro curies in single sitting. portal.
•• Therapeutic uses- Graves’ disease, treat secondaries after
Carcinoid syndrome:
surgery in thyroid differentiated cancers.
•• Vasomotor symptoms (80%: Cutaneous flushing is the most
102. Ans. (a) Scalenus anterior and medius common symptom**
Ref Surgery Sixer 4th Edition page 205 •• GIT: Explosive diarrhea. Due to serotonin.
•• Cardiovascular: The three most common cardiac lesions
Potential area of compression of Subclavian Artery
are pulmonary stenosis (90%), tricuspid insufficiency
•• Interscalene space: Anteriorly – Scalenus anterior,
(47%), and tricuspid stenosis (42%)***
posteriorly – Scalenus medius, inferiorly – 1st rib
•• Asthmatic attacks: Bronchospasm
•• Intercostoclavicular space: Between 1st rib and clavicle
•• Malabsorption and pellagra (dementia, diarrhea, dermatitis)
•• In the axilla: Between pectoralis minor tendon attaching to
due to excessive diversion of tryptophan.
coracoids process
•• Types of flushing: Diffuse erythematous, violaceous,
Cervical rib passed between scalenus anterior and medius and
prolonged flushes and bright-red patchy
compresses the Subclavian artery and lower trunk of brachial
Most common type of flushing seen in gastric carcinoids:
plexus
Bright-red patchy. (Not in Ileal Carcinoids)
Octreotide reduces these symptoms
Stress and a huge meal increase these symptoms
LXIII
Surgery Sixer for NBE
105. Ans. (d) Peterson Hernia •• Small bowel ischemia usually presents as lack of wall
Ref: Surgery Sixer 4 Edition Page 507
th enhancement, dilation and in more advanced cases,
pneumatosis intestinalis and portal venous gas. The CT
•• Spigelian hernia- Via the Arcuate line infra umbilical
scan finding explained here is classical NOMI.
•• Richter’s hernia- a part of circumference is obstructed
Mesenteric Ischemia is discussed under three broad headings:
(Diarrhea+)
•• Acute Mesenteric artery Ischemia
•• Peterson Hernia- Hernia behind Roux Limb
•• Chronic mesenteric Ischemia
•• Stammers Hernia- Hernia behind Transverse mesocolon
•• Mesenteric Vein Thrombosis
•• Ogilvie’s Hernia- Via a defect in conjoint tendon
•• Non-Occlusive Mesenteric Ischemia (NOMI)
•• Laugier’s Hernia- Via defect in lacunar ligament
•• Littre hernia- Meckel’s diverticulum containing hernia Etiology:
•• Amyand hernia- Appendix containing hernia •• Sudden occlusion of superior Mesenteric Artery [50%]
•• Berger’s Hernia- hernia into Pouch of Douglas Grynfeltt Atherosclerosis (Most common cause of Chronic
Hernia- Superior lumbar hernia Mesenteric Ischemia)
•• Petit Hernia- Inferior lumbar hernia. Embolism(Most common cause of Acute SMA ischemia
•• Narath hernia- Hernia seen in congenital dislocation of hip 90% cases)**
•• Maydl’s hernia- W’ shaped loop in hernial sac. Retrograde Vasculitis (Poly arteritis nodosa)
strangulation occurs Fibromuscular dysplasia
•• Cloquet’s hernia- The sac lies under the fascia covering the •• Mesentric vein occlusion [25%]
pectineus muscle. Thrombosis due to OCP, Polycythemia, Neoplasm
•• Gibbon’s hernia- Hernia with hydrocele infiltrating
•• Beclard’s hernia- Femoral hernia via opening in Saphenous •• Nonocclusive obstruction [25%]
Vein** Severe shock
Acute Mesenteric Ischemia:
106. Ans. (a) Immediate appendectomy
Clinical features:
Ref: Sabiston 20th Edition Page 1306-1307
•• Sudden severe abdominal pain, vomiting, abdominal
Pregnancy and Appendicitis distension
•• Typical presentation seen in 50-60% cases. •• Central abdominal pain with symptoms out of proportion
•• Symptoms are nausea, vomiting and febrile response which to signs present* (Hall Mark feature**)
can be seen in normal pregnancy also. •• Functional obstruction with absent bowel sounds
•• Appendix is pushed more cephalad by gravid uterus. •• Shock and peritonitis are rapid
•• MC trimester of appendicitis- 2nd trimester. •• 100% mortality if untreated.
•• Some lab values like Leukocytosis, CRP increase may be •• In emboli cases the obstruction is in mid to distal SMA,
physiological also. distal to origin of middle colic artery. In such cases some
•• High rate of negative appendectomy in pregnant is seen(25- part of bowel is viable. (Fig)
50%) due to all the above reasons. •• In thrombosis cases the obstruction is at origin of SMA. In
•• Risk of preterm labour and fetal loss is seen in appendicitis such cases the bowel is gangrenous from DJ flexure to right
cases and is less in uncomplicated appendicitis. colon.
•• IOC – USG done in left posterior oblique or left lateral •• Investigation to diagnose- Laparotomy**
decubitus position. (Sensitivity- 78% and specificity -83%) Treatment
•• MRI without Gadolinium has excellent sensitivity and •• Resection of non-viable bowel with its mesentery
specificity. •• Second look operation
•• In patients for whom MRI is normal needs no appendectomy
and is shown to be cost effective.
•• If MRI is not available- CT scan can be done, as by 2nd and
3rd trimester organogenesis has already happened.
•• Laparoscopic appendectomy is current recommendation,
but Hasson Open technique is preferred for primary port to
avoid injury to gravid uterus.
LXIV
Latest Questions Paper 2020-2019
Chronic Mesenteric Ischemia: •• The resection with drainage procedures are all complicated
•• Presents with intestinal angina, weight loss, chronic diarrhea surgeries and the nutrition will be started by a feeding
•• Gold standard for chronic mesenteric ischemia- Mesenteric jejunostomy Tube which will be inserted during the surgery.
angiography. •• The purpose of feeding jejunostomy is, even if there is a leak
•• Atherosclerosis in main splanchnic artery (coeliac, IMA, from pancreatic jejunostomy anastomosis we can feed the
SMA) is the most common cause of chronic mesenteric patient as we have kept the tube distal to the anastomosis.
ischemia.
109. Ans. (d) Replace drain amount with IV Ringers lactate
Ref: Sabiston 20th Edition Page 1548
Recent Advances Pancreatic leak or pancreatic fistula, which has been defined by
•• SQUID- superconducting quantum interference device (SQUID) the International Study Group on Pancreatic Fistula as:
magnetometer to detect mesenteric ischemia noninvasively •• “output via an intraoperatively placed drain (or percutaneous
drain) of any measurable volume on or after postoperative
Mesenteric vein thrombosis: day 3, with amylase >3 times normal serum value,”
•• Mesenteric vein thrombosis is usually due to small •• Incidence- 5- 22%
peripheral vein thrombosis** [SMV OR IMV thrombosis
110. Ans. (b) Trophozoite
is rare]
•• Hence thrombectomy is not needed** (not indicated) Ref: Sabiston 20th Edition Page 1451
•• Fluid resuscitation •• Cysts ingested in contaminated food reaches the stomach-
•• Heparin anticoagulation is sufficient intestine and in intestine the cysts release trophozoites
•• If peritoneal signs are present- urgent laparotomy needed which enter and invade the colon.
•• If peritoneal signs absent- Heparin [5 days], oral •• Trophozoites reach the liver via portal vein. It does not
anticoagulation lifelong along with bowel rest and fluids is invade via lymphatics.
enough. •• It causes necrosis of tissues in liver. The cavity will have
acellular proteinaceous debris surrounded by a rim of
Amoebic trophozoites in wall.
•• Leucocytes will be there initially but later lysed by
Colonic Ischemia: Trophozoites.
•• Most common site of Ischemic Colitis- Splenic Flexure
(Griffith Point) 111. Ans. (a) Early Enteral feeds
•• Watershed area in colon- Splenic Flexure Ref: Bailey and Love 27th Edition Page 1225
•• Most common site of Ischemic Stricture- Sigmoid Colon.
(Sudeck’s Point) •• In management of acute pancreatitis, previously TPN
was the treatment of choice. But TPN was associated
with increased bacterial translocation in GUT and hence
Section D: Hepatobiliary and Pancreatic the enteral feeding is now supported if patient is having
sufficient BP.
•• Enteral feeding is suggested by nasogastric tube and if
108. Ans. (c) Feeding Jejunostomy
patient has vomiting or unable to tolerate NG feeding, we
Ref: Practical Question (Based on Surgery Sixer 4th Edition) go with Nasojejunal tube.
•• As Surgical Gastroenterologist we do many complicated
112. Ans. (c) Severe
procedures for Chronic pancreatitis like mentioned below:
Ref: Bailey and Love 27th Edition page 1200
Surgeries:
Tokyo Consensus Guidelines for Severity grading of Acute
Drainage Procedures Resection+ Drainage Cholecystitis:
Procedures Grade III (Severe) Acute Cholecystitis:
1. F or dilated duct: >7 mm With head mass: Associated with any one of the following organ dysfunction:
diameter: •• CVS: Hypotension requiring Dopamine or Noradrenaline
Modified Puestow 1. Whipple’s procedure (not •• Neurological: Decreased consciousness level
procedure done nowadays) •• Respiratory: Pa02/Fi02 ratio <300
•• Renal dysfunction: Oliguria: creatinine >2 mg/dl
2. For Undilated duct: 2. Duodenum Preserving •• Liver dysfunction: PT – INR elevated
Izbicki Procedure in which Pancreatic head resection •• Hematological Dysfunction: Platelet count <1 lakh/mm3
V- Shaped excision of the (DPPHR) Grade II (Moderate cholecystitis)
duct done. Beger’s Procedure •• Elevated WBC count >18000/mm3
Bern’s Modification •• Palpable tender mass in right upper quadrant.
Frey’s Procedure •• Duration of complaints >72 hours.
•• Marked local inflammation (Gangrene, Abscess, Biliary
peritonitis, Emphysematous Cholecystitis)
LXV
Surgery Sixer for NBE
Grade I (Mild Acute Cholecystitis) •• They are typically on the middle cerebral artery junctions
•• Grade 1 has none of the above features. with the small perforating lenticulostriate vessels, leading
•• Health person with mild inflammations only. to hemorrhage into the putamen.
•• Cholecystectomy is a safe and low risk procedure. Clinical Features:
In Grade III high risk cases sometimes if patient is unfit for •• Stroke features
a cholecystectomy a percutaneous cholecystostomy can be •• Symptoms are more likely to include headache than
performed by Radiologist. ischemic stroke.
Diagnosis:
Section E: Specialty Surgery •• The diagnosis is with CT, usually done in an emergency
department setting.
113. Ans. (c) Lymphoma •• Further investigation is generally done with contrast MRI
Ref: Bailey and Love 27th Edition Page 935 or magnetic resonance angiography. Any suggestion of
aneurysm or AVM is followed by conventional catheter
Bailey explains with an illustrated picture of the various
angiography.
mediastinal masses.
The picture shows clearly a superior mediastinal mass:
The various superior mediastinal masses are
•• Lymphoma
•• Thyroid Extension
•• Parathyroid
Essential Clinical Features: •• Patient put in left lateral position with affected lung up.
•• MC symptom is Hemoptysis (<50% cases) •• Incision made below scapula tip.
•• Ominous feature- Pleural fluid with blood •• Nowadays lung cancer surgery done by VATS.
•• Small cell cancer has Eaton Myasthenic Lambert
•• Surgery with Curative intent is possible up to T1- T3 tumors
Syndrome**.
and N0-N1 cases only.
•• Pancoast tumor is a feature of squamous type- causes
•• For Non-small cell lung cancer surgery is done with CURATIVE
Horner’s Syndrome
INTENT, even T3 tumors involving chest wall is resected en
•• SVC syndrome is a feature of small-cell cancer.
bloc.
TNM staging of Lung Cancer(Simplified version)
Types of Surgery for Lung Cancer:
T Staging
•• T1- <3 cm Segmentec- Lobectomy Pneumonec- Broncho-
•• T2- 3-5 cm or involves main Bronchus, involves tomy and tomy plastic Lung
Visceral Pleura or having lung Atelectasis Wedge resection
•• T3- 5-7 cm or involves chest wall, parietal pleura, resection
phrenic nerve, parietal pericardium and superior
•• For Small •• Early stage •• Removal of •• To avoid
sulcus tumors
tumors lung cancer whole lung. compli-
•• T4- >7 cm lesion or involves diaphragm, heart, great
•• Ligate the this is the •• Indicated for cations
vessels, esophagus, vertebra, carina, recurrent
segmental Treatment centrally lo- related to
laryngeal nerve.
pulmo- of Choice. cated tumors lung resec-
N Staging
nary •• The Pul- or those tion, lung
•• N1- Ipsilateral peribronchial and hilar nodes.
artery and monary tumors cross- tissue pre-
•• N2- Ipsilateral mediastinal and subcarinal nodes
vein and artery and ing the main servative
•• N3- Contralateral any node.
bronchus. vein with fissures. operation is
M Staging
bronchus •• Main Pulmo- done.
•• M1a- Tumor with contralateral lobe nodules, pleural
supplying nary artery •• Sleeve lung
or pericardial nodules.
the lobe is and superior resection
•• M1b- Single Extra Thoracic Mets
ligated. and inferior of centrally
•• M1c- Multiple Extra Thoracic Mets in one or more
•• 2-3% mor- pulmonary located
organs.
tality vein ligated. tumor with
Investigations Main bron- reanasto-
•• X-ray chest diagnoses most cancer chus is di- mosis of
•• 1st investigation in suspected lung cancer- CECT Thorax. vided. cut major
•• Drainage of Bronchus
(>2 cm nodes are mostly metastatic)
the space is to remain-
•• IOC to look for mets- PET CT
done without ing lobar
•• Sputum cytology- may reveal malignant cells but false
suction. Air bronchus.
negative rate is high.
in the space
Biopsy Procedures: is gradually
•• Centrally- located tumors- Bronchoscopy and replaced by
Transbronchial Needle Aspiration (TBNA) fluid level.
•• Peripherally- located tumors- Percutaneous CT guided
Biopsy. Complications of Lung Resection:
Surgical Staging: •• Bleeding
•• Respiratory infection
•• In cases of doubtful involvement of nodes direct nodal
•• Persistent air leak
biopsy done by three methods
•• Broncho pleural fistula- Most serious Complication. Higher
•• Mediastinoscopy via a 2 cm incision above suprasternal
Morbidity and Mortality seen.
notch.
•• Mediastinotomy via an incision in 2nd ICS on affected side. Prognosis of Lung Cancers:
•• VATS biopsy of nodes Five-year survival rate
Surgical Treatment: •• Squamous cell cancer- 35-50%
•• Adenocarcinoma- 25-45%
•• Thoracotomy – Most frequent indication of Thoracotomy
•• Small cell cancer- 0-5%
is lung cancer.
•• Double lumen Endotracheal tube inserted and operative 116. Ans. (a) Congenital hydrocele
side lung collapsed.
Ref: Surgery Sixer 4th edition page 752
LXVII
Surgery Sixer for NBE
•• The picture clearly shows the communication of the are components of the same pathological process called
peritoneal cavity to the root of scrotum. Hence the fluid superficial fibromatosis
collects in the entire sac.
•• Typical inverted ink bottle effect is seen in this case of
hydrocele.
LXVIII
Latest Questions Paper 2020-2019
124. Ans. (a) Palm rule; (b) Lund and browder; (c) Wallace rule;
(d) Berkow Formula
Ref: Bailey and Love 27th Edition Page 621
•• Parkland Formula is used to calculate fluid to be infused.
120. Ans. (a) Allows re introduction of Displaced tube; 128. Ans. (c) Moves poorly with respiration
(b) A type of tracheostomy flap; (c) avoid 1st tracheal ring; Ref: Bailey and Love 27th Edition Page 1229
(d) Free edge of the Bjork flap is sutured to the Transverse •• Pseudocysts usually arises from retroperitoneum and hence
skin incision less moves with respiration.
Ref: Bailey and Love 27th Edition Page 746 •• Fluid thrill is absent like in a hydatid cyst.
•• This is an Inferiorly based flap •• Fluctuation is absent as it is retroperitoneal
•• Upper border cannot be felt and it can be below the xiphoid
121. Ans. (a) Can occur after blunt trauma to lower chest; process.
(d) The defect is mostly large enough to cause symptoms;
(e) Penetrating injury below 5th Intercostal space can cause 129. Ans. (a) Ann Arbor staging; (b) Cotswold staging; (c)
diaphragmatic injury Lugano staging
Ref: Bailey and Love 27th Edition page 391 Ref: Various sources
•• VATS or Laparoscopy is the ideal method to diagnose the The staging system used for Lymphoma are:
diaphragm injuries •• Ann Arbor Staging
•• Diaphragmatic rupture is approached via abdominal •• Cotswold Staging
approach •• Lugano Staging (For Primary GI lymphoma)
122. Ans. (a) Murphy sign; (b) Boas sign 130. Ans. (b) Torsion; (c) Epididymo orchitis
Ref: Bailey and Love 27th Edition Page 1199 Ref: Bailey and Love 27th Edition Page 1500
•• Pointing and Obturator sign are seen in Acute appendicitis Acute testicular pain causes:
•• Torsion
•• Epididymo orchitis
LXIX
Surgery Sixer for NBE
131. Ans. (a) Seminomas spread via lymphatics; (b) Lymphatic Long saphenous vein (Most commonly used.
drainage is to Para aortic nodes; (d) Tumor markers are Short saphenous vein
elevated Cephalic veins
Left Internal mammary artery (LIMA.
Ref: Bailey and Love 27th Edition Page 1508
Bilateral IMA (BIMA.
•• Testicular tumors will not spread to inguinal nodes unless Radial artery
scrotal violation is done.
•• Transscrotal orchiectomy is not advised, we must do 138. Ans. (b) On upper head of epididymis; (e) Up and backward
Inguinal orchiectomy. on testis
Ref: Bailey and Love 27th Edition Page 1504
132. Ans. (a) Rectal Decompression; (b) Pelvic Colon was
rotated against adhesion; (d) Show anticlockwise rotation Spermatocele:
Ref: Bailey and Love 27th Edition Page 1295 •• Unilocular retention cyst derived from sperm conducting
•• Thumb print sign is seen in Ischemic Colitis* part of epididymis
•• Sigmoid Volvulus has more Anticlockwise rotation than •• Lies in epididymal head typically behind upper pole of testis
clockwise rotation. •• Soft and lax
•• Transillumination + as per Bailey
133. Ans. (a) Opisthorchis viverrini; (b) Clonorchis sinensis •• Fluid resembles – Barley water appearance containing
Ref: Bailey and Love 27th Edition Page 1208) spermatozoa*
•• Larger ones are excised. Smaller ones observation
Risk of cholangiocarcinoma:
Parasitic infections: 139. Ans. (b) Absent Breast unilaterally or Bilaterally may
•• Opisthorchic viverrini be seen; (c) Absent sternal portion of Pectoralis major;
•• Clonorchis sinensis (d) More common in males
134. Ans. (a) Cheatle cut; (c) Side to side Ref: Bailey and Love 27th Edition Page 865
Ref: Bailey and Love 27th Edition Page 97 Poland syndrome:
•• Cheatle cut is a linear cut made in the bowel with smaller •• MC in males
diameter for an end to end anastomosis. •• Congenital absence of Pectoralis major and breast
135. Ans. (a) Liver cirrhosis; (b) Digitalis; (c) Leprosy; (d) 140. Ans. (b) Palpable of gallbladder; (c) Peripancreatic
Klinefelter syndrome carcinoma; (d) Obstruction in common bile duct
Ref: Bailey and Love 27th Edition Page 882 Ref: Surgery Sixer 4th Edition Page 549
•• Gynecomastia is not associated with renal failure. •• Courvoisier law states Gallbladder is palpated in malignancy
•• Digitalis, ketoconazole, Estrogen drugs can cause and not in stones.
gynecomastia •• It is due to obstruction of CBD
136. Ans. (a) It is potentially curative; (b) Impotency is the 141. Ans. (a) Thymoma; (b) Pancoast tumor of right lung apex;
most common side effect; (c) Can lead to severe stress (e) Retrosternal goitre
incontinence; (e) Walsh modification preserves erectile Ref: Bailey and Love 27th Edition Page 926
function
SVC syndrome- Superior vena caval obstruction or
Ref: Bailey and Love 27th Edition Page 1473 compression is caused by
Radical prostatectomy: •• Retrosternal goitre
•• Pancoast tumor of Right lung apex
•• Suitable for Localised disease
•• Thymoma
•• Done if life expectancy >10 years
•• High incidence of impotence 142. Ans. (b) Due to 2nd brachial arch remnant; (d)
•• Low incidence of severe stress incontinency Transillumination may be seen; (e) In cases of sinus inner
•• Procedure involves removal up to seminal vesicles. tract is at tonsillar fossa
•• Bladder neck is reconstructed and connected to urethra.
Ref: Bailey and Love 27th Edition Page 754
•• Walsh modifications preserves Neurovascular bundle and
preserves erectile function in 60% •• Branchial cyst is due to vestigial remnant of 2nd branchial
cleft.
137. Ans. (b) Great saphenous vein; (d) Right internal mammary •• The fistula may be unilateral or Bilateral
artery; (e) Left Internal mammary artery •• External orifice is at junction of middle and lower border
Ref: Bailey and Love 27th Edition Page 890 of SCM
•• Inner opening is at tonsillar fossa
•• The vessels used in CABG are:
LXX
Latest Questions Paper 2020-2019
143. Ans. (b) Small intestine recover faster than colon; (d) 150. Ans. (a) Should be given according to sensitivity of the
vomiting and abdominal distension seen spectrum of...; (c) Maximum blood and tissue concentration
Ref: Bailey and Love 27th Edition Page 1285 of antibiotic...; (d) Should be repeated if surgery is
prolonged
•• Paralytic ileus recovers in following order- Small intestine
1st - Colon next and Stomach as last Ref: Bailey and Love 27th Edition Page 74
•• MC cause is hypokalemia Prophylactic Antibiotics:
•• Maximum Blood and tissue levels should be present at the
144. Ans. (a) Associated with tuberous sclerosis; (b) Has high fat
time at which the first incision is made.
content in CT; (d) Wunderlich syndrome is seen; (e) Partial
•• Prophylactic antibiotic must be given 1 hour before Skin
nephrectomy done
incision.
Ref: Bailey and Love 27th Edition Page 1416 •• Repeated dose given at every 4 hours if surgery is prolonged
•• Wunderlich syndrome—Sudden rupture of the tumor and and discontinued maximum in 24 hours.
leaking of the blood in retroperitoneum causes Hemorrhage.
151. Ans. (a) Radical prostatectomy; (c) Radical radiotherapy
PGI MAY 2019 Ref: Bailey and Love 27th Edition Page 1474
Intermediate Risk Prostatic cancer:
145. Ans. (b) Increases blood flow to wound which helps in •• Younger , fitter men < 70 Years- Radical prostatectomy or
healing (d) Compression by a foam cells dressing covered Radical Radiotherapy
with a non... (e) Compression by a foam cells dressing •• Elderly patients- Active monitoring or if outflow
covered with a non- obstruction- Transurethral resection with or without
Ref: Bailey and Love 27th Edition Page 30 Hormone therapy advised.
•• Vacuum Assisted Closure device stimulates formation of
152. Ans. (b) T2a; (d)N1
granulation tissue
•• The negative suction pressure of -125 mm Hg is used. Ref: Bailey and Love 27th Edition Page 1471
•• T2a- On rectal examination the tumor confined within
146. Ans. (b) Ethilon-Nylon prostatic capsule and involving one lobe. (T2b- involves
Ref: Bailey and Love 27th Edition Page 114 both lobes)
•• Ethilon or Nylon is a non absorbable suture* •• N1- Node +
•• Polyglactin is a longest absorbable suture material.
153. Ans. (a) Transverse skin crease incision; (c) Centered on
147. Ans. (c) Deep vein thrombosis (e) Lymphedema Mc Burney point; (d) Centered on the midclavicular–mid-
inguinal line
Ref: Bailey and Love 27th Edition Page 998
Ref: Bailey and Love 27th Edition Page 1309
•• CCF, Hypoalbuminemia and Myxedema causes bilateral
Leg swelling •• Lanz incision is a Skin incision
•• Horizontal , cosmetic incision
148. Ans. (a) Joll’s thyroid retractor; (e) Golingher retractor
154. Ans. (a) Hematochezia (d) Intestinal obstruction; (e)
Ref: Bailey and Love 27th Edition Page 1582 Altered bowel habits
The following are the self Retaining Retractor: Ref: Bailey and Love 27th Edition Page 1264
•• Joll Retractor- Thyroid
•• Goligher Retractor- Sigmoid colon and rectum •• Right colon cancer- Produces Melena and Anemia
•• St Mark’s Perineal retractor- for APR •• Left colon cancer produces obstruction
•• Traver’s Retractor- For intermediate type operations like
155. Ans. (c) T4; (d) N1
Herniorrhaphy, Varicose veins etc
•• West Retractor- for Minor procedures Ref: Bailey and Love 27th Edition Page 1262
•• T4- Breaches Serosa or another organ directly
149. Ans. (a); Caused by mechanical blockage by impacted •• N1- 1-3 nodes involved.
stones; (d) Associated with enterobiliary fistula; (e)
Treatment require surgery in all cases 156. Ans. (a) Most commonly seen in elderly men
Ref: Bailey and Love 27th Edition Page 1220 Ref: Bailey and Love 27th Edition Page 1322
•• Mirizzi syndrome is CBD compression by GB stone. •• Complete Rectal Prolapse (Procidentia)
•• It’s a mechanical block of Small bowel by Gallstone which •• Mc in women and associated with another pelvic organ
has entered via an enterobiliary fistula. prolapse
•• Surgery is indicated always. •• Usually > 4cm prolapse.
•• When large may contain small intestine or Bladder.
LXXI
Surgery Sixer for NBE
157. Ans. (c) Flushing is done on the abdominal surface; (d) 164. Ans. (a) Varicosities on posterolateral aspect of leg is due to
Effluent is usually solid accessory saphenous vein
Ref: Bailey and Love 27th Edition Page 1278 Ref: Bailey and Love 27th Edition Page 972
•• Colostomy can be a temporary or permanent procedure. •• Varicosities are not due to blockade of Superficial veins like
•• Made in Epigastric or Left iliac fossa regions. SSV or GSV, it is due to blockage of Deep veins.
•• Postero lateral varicosities is due to Accessory Saphenous
158. Ans. (b) Aspiration ; (c) Puncture; (d) Injection
vein.
Ref: Bailey and Love 27th Edition Page 1169
•• PAIR means Percutaneous Aspiration Infusion of Scolicidal 165. Ans. (a) Flight journey more than 6 hours is a risk factor;
agents and Respiration (It is done with 20% Hypertonic (c) Tenderness can be elicited in calf muscles on applying
saline, 0.5% silver nitrate solutions pressure; (d) Low grade fever with increased pulse rate
Ref: Bailey and Love 27th Edition Page 987
159. Ans. (a) Smoking; (c) GERD; (d) Barrett’s esophagus; (e)
•• DVT is more common in Calf veins and can be unilateral
Obesity
or Bilateral.
Ref: Bailey and Love 27th Edition Page 1085 •• Bilateral in 30%
•• Adenocarcinoma is a risk factor of Squamous cell cancer •• Low grade pyrexia is seen when pulmonary embolus is seen.
only. •• Pulmonary embolus occurs in 10% cases.
•• Affected vein is not seen as cord except stiff calf and
160. Ans. (b) Commonly associated with Downs syndrome; (e) tenderness along course of deep veins
Treated by surgery
Ref: Bailey and Love 27th Edition Page 1219 166. Ans. (b) Mesh can be braided; (c) Mesh can be attached to
defect by glue or suture or staples; (e) Shrinkage of mesh
•• MC associated with Duodenal stenosis or atresia .
may occur after some years
•• Duodenal obstruction is at 2nd part.
•• Duodenoduodenostomy is TOC. Ref: Bailey and Love 27th Edition Page 1028
•• If occurs in later life , with pancreatitis- head resection alone •• Usually Non absorbable meshes are used.
is enough. •• Mesh is kept over the defect to overlap the defect at least
2 cm but up to 5 cm if possible.
161. Ans. (a) Hirschsprung’s disease; (b) Cystic fibrosis
Ref: Bailey and Love 27th Edition Page 60 167. Ans. (c) Rotation flap; (e) Bilobed flap
•• Toxic megacolon is associated with Ulcerative colitis, Ref: Bailey and Love 27th Edition Page 641
Crohn’s disease, Clostridium infection, Amoebic infection The flaps used to close a convex defect are
•• Hirschsprung disease causes only megacolon and no •• Rotation flaps
toxicity. •• Bilobed Flap
162. Ans. (a) Most commonly seen posteriorly in the midline; 168. Ans. (a) Resolves spontaneously; (e) Usually disappears at
(b) Commonly due to local trauma, vaginal delivery, anal 1yr of age
sex...; (c) Lateral internal sphincterotomy is the surgical Ref: Bailey and Love 27th Edition Page 613
procedure of choice; (e) Conservative management include
•• Salmon Patch is seen in Forehead or nape of neck.
dietary modifications
•• Appear at birth and disappear at 1 year
Ref: Bailey and Love 27th Edition Page 1352
•• Special feature of Anal fissure is, it won’t extend proximal 169. Ans. (b) Anastomosis b/w stomach and 1st part of
to dentate line. duodenum
•• Notaras Lateral sphincterotomy is the surgical treatment of Ref: Bailey and Love 27th Edition Page 1120
Choice. •• Billroth 1- Gastro Duodenal anastomosis
•• Billroth II- Gastro Jejunal anastomosis
163. Ans. (a) Almost always associated with double ureter; (d)
Open distally beyond the normal opening 170. Ans. (a) Pain
Ref: Bailey and Love 27th Edition Page 1401 Ref: Bailey and Love 27th Edition Page 28
Duplex Ureters: •• Early sign is pain on passive stretching
•• Incidence- 1 in 50 •• Late sign is absent pulses
•• Unilateral more common
171. Ans. (a) No color change; (e) Diameter less than 5 mm
•• MC among girls
•• Can be complete or incomplete type Ref: Bailey and Love 27th Edition Page 610
•• Associated with VUR in children •• Asymmetry
•• Weigert Meyer rule- It opens Distally and medially. •• Border irregularity
LXXII
Latest Questions Paper 2020-2019
LXXIII
Surgery Sixer for NBE
184. Ans. (d) 3rd Degree burns •• It is a rapid spreading synergistic infection, MC organism is
Ref: Surgery Sixer 4th Edition Page no 873 Group A beta hemolytic Streptococci**
•• If seen in Scrotum is known as Fournier’s Gangrene**
•• Third degree burns also known as full thickness burns is
characterized by Burns extending up to subcutaneous tissue 192. Ans. (c) NCCT
involving all layers.
Ref: Surgery Sixer 4th Edition Page no 88
•• Eschar is seen in 3rd degree burns
•• In head injury patients, the immediate investigation done is
185. Ans. (c) Non Suppurative and Invasive Non Contrast CT scan.
Ref: Surgery Sixer 4th Edition Page no 35
•• Cellulitis is the non suppurative infection involving the Section B: Head and Neck, Thyroid and Breast
Subcutaneous tissue and deeper planes.
•• Erysipelas is another non suppurative infection involving
193. Ans. (b) Total thyroidectomy
only Skin and Subcutaneous tissue.
•• Erysipelas will involve the ear , whereas Cellulitis will not Ref: Surgery Sixer 4th Edition Page 228
attack the ear. This sign is known as Milian’s Ear Sign. •• Papillary cancer- Total thyroidectomy + Modified Radical
neck node dissection if nodes are involved.
186. Ans. (a) Fat embolism •• Follicular neoplasm- Hemithyroidectomy followed by
Ref: Surgery Sixer 4th Edition Page no 51 Completion Total thyroidectomy if specimen comes as
•• One of the dangerous Complication of Long Bone Fractures malignant.
is Fat Embolism**
194. Ans. (c) Excision
187. Ans. (b) Ultrasound Ref: Surgery Sixer 4th Edition Page 188
Ref: Surgery Sixer 4th Edition Page no 543 •• Ranula is a mucus Extravasation cyst of sublingual salivary
•• The History is suggestive of Gall stone disease- hence the gland.
next investigation is USG •• Surgical management: Excision of cyst and affected gland.
•• USG is sensitive in 90% cases and is the IOC for Gall stone •• Incision and drainage results in Recurrence
disease.
195. Ans. (b) Below Inferior Constrictor
188. Ans. (a) Tongue Fall Ref: Surgery Sixer 4th Edition Page 312
Ref: Surgery Sixer 4th Edition Page no 82 •• Kilian’s Dehiscence is a defect in Inferior constrictor of
•• In trauma, the first maneuver done is Jaw thrust and Chin Pharynx between Thyro and Cricopharynx muscles.
lift to prevent Tongue falling Backwards and causing •• This is the defect through which Zenker’s Diverticulum
Obstruction of airway, happens.
189. Ans. (d) Glasgow coma scale 196. Ans. (a) Squamous cell cancer
Ref: Surgery Sixer 4th Edition Page no 83 Ref: Surgery Sixer 4th Edition Page no 167
The Question is asking about the primary survey; •• In Oral cancer the most common histology is Squamous cell
•• Airway cancer.
•• Breathing
197. Ans. (a) Luminal A
•• Circulation
•• Disability Ref: Surgery Sixer 4th Edition Page no 267
•• Exposure •• There is a molecular classification of Cancer breast based on
According to that Glasgow score is made after stabilization of Immuno-Histochemistry
patient. •• The classification is based on ER, PR and HER-2 Status
•• Luminal A is the most common type- ER+, PR+, HER-2
190. Ans. (a) Triage negative
Ref: Surgery Sixer 4th Edition Page no 81
•• In mass casualty, Triage is done to sort out the patients Section C: GIT
according to severity
•• Red card is given to patients who need utmost priority and
need management in 30 minutes, 198. Ans. (d) Malrotation of Gut
•• Yellow 2nd and Green 3rd Priority. Ref: Surgery Sixer 4th Edition Page 419
•• Newborn Baby with Bilious Vomiting is seen in Duodenal
191. Ans. (c) Necrotizing fasciitis
Atresia and Malrotation.
Ref: Surgery Sixer 4th Edition Page no 35
LXXIV
Latest Questions Paper 2020-2019
•• Since the baby is having multiple air fluid levels- this is not 207. Ans. (a) Sister Mary Joseph Nodule
duodenal atresia. Ref: Surgery Sixer 4th Edition Page no 361
•• Duodenal atresia will have double bubble appearance.
•• Carcinoma stomach cases will have a malignant deposit in
199. Ans. (a) Mucoviscidosis Umbilicus level by transcoelomic spread known as Sister
Mary Joseph nodule.
Ref: Surgery Sixer 4th Edition Page 418
•• Mucoviscidosis is the other name of Cystic fibrosis . 208. Ans. (b) Bile salts.
•• Earliest manifest of Cystic Fibrosis is Meconium ileus. Ref: Surgery Sixer 4th Edition Page no 407
200. Ans. (b) NG tube insertion and CXR to check position of •• In terminal ileum , Enterohepatic Circulation(EHC) of Bile
tube salts is seen.
•• The Bile salt pool is maintained by the EHC in which the
Ref: Surgery Sixer 4th Edition Page 316
bile salts entering from liver to GIT is brought back to the
•• H/o of frothiness in mouth, drooling of saliva and dyspnea- Liver.
gives us a suspicion on Congenital Tracheo Esophageal
Fistula or a Congenital Diaphragmatic hernia (CDH) 209. Ans. (a) Boerhaave syndrome
•• Coiling of Ryles tube is one method to find out the pathology Ref: Surgery Sixer 4th Edition Page no 318
as Esophageal atresia.
The triad of Boerhaave syndrome (Spontaneous Rupture of
•• Ryles tube entering the Chest is a finding in CDH.
Esophagus- Barotrauma)
201. Ans. (a) Chemoradiation Mackler’s Triad:
Ref: Surgery Sixer 4th Edition Page no 489 •• Vomiting
•• The regimen used for Anal Canal cancer is NIGRO Regimen •• Chest pain
•• Chemo radiation- 5FU+ Mitomycin chemotherapy + •• Subcutaneous emphysema
Radiotherapy is used for Anal SCC. There will be Naclerio V sign known as Pneumomediastinum
202. Ans. (a) No occurrence after surgery 210. Ans. (b) Rectum is involved
Ref: Surgery Sixer 4th Edition Page no 412 Ref: Surgery Sixer 4th Edition Page no 412
•• Crohn’s Disease •• Crohn’s disease involves from Mouth to anus Except
•• Incurable Rectum**
•• May occur from Mouth to anus •• Transmural involvement seen
•• Only Rectal area is spared. •• Recurrence is common
•• Fistula and Fissure are common
203. Ans. (b) pH metry
211. Ans. (b) Early Dumping Syndrome
Ref: Surgery Sixer 4th Edition Page no 308
Ref: Surgery Sixer 4th Edition Page no 356
•• 24 hours Ambulatory pH monitoring is used for GERD and
not in cancer. •• Early Dumping is due to Hypovolemia due to Fluid Shift,
happens in 30 minutes
204. Ans. (a) Ochsner Sherren regimen •• Late Dumping is due to Hypoglycemia , happens after 2
Ref: Surgery Sixer 4th Edition Page no 424 hours
•• For appendicular mass the conservative regimen used is 212. Ans. (b) Betel Chewing
known as Ocshner-Sherren Regimen.
Ref: Surgery Sixer 4th Edition Page no 320
205. Ans. (d) Colocolic Intussusception •• Betel Chewing is associated with Oral malignancies.
Ref: Surgery Sixer 4th Edition Page no 418
213. Ans. (b) Leukocytosis
•• The sign shown here is known as CLAW sign* in Barium
Ref: Surgery Sixer 4th Edition Page no 424
enema
•• Another appearance is Coiled Spring Appearance* •• In ALVARDO SCORE- all have 1 point except – Leucocytosis
and Tenderness in Left iliac fossa.
206. Ans. (d) Refractory Fistula
Ref: Surgery Sixer 4th Edition Page no 412 Section D: Hepato Biliary and Pancreas
•• Fistula is very uncommon in Ulcerative Colitis and seen in
Crohn’s Disease only. 214. Ans. (a) Hemorrhage
•• Refractory fistula in Crohn’s disease is treated by Infliximab.
Ref: Surgery Sixer 4th Edition Page no 678
LXXV
Surgery Sixer for NBE
217. Ans. (a) Neuralgia 224. Ans. (b) Surgery after 5 years.
Ref: Surgery Sixer 4th Edition Page 838 Ref: Surgery Sixer 4th Edition Page no 513
•• Nerve injury is the most common serious complication in •• This is a picture showing Umbilical Hernia- the Anatomical
Stripping. repair is done in 5 years of age.
•• Injury to saphenous nerve, sural nerve, common peroneal •• Usually Umbilical hernia in paediatric age will not go for
nerve obstruction.
220. Ans. (b) Gut Ref: Surgery Sixer 4th Edition Page no 142)
Ref: Surgery Sixer 4th Edition Page 144 •• The transplanted kidney is usually placed in Iliac Fossa and
most surgeons prefer Right Iliac fossa as the left fossa has
•• Highest chances of Graft rejection and Graft versus Host Sigmoid Colon
disease is seen in Small Bowel Transplant •• The Left Kidney is usually preferred as it has long left renal
vein.
LXXVI
Latest Questions Paper 2020-2019
LXXVII
Surgery Sixer for NBE
Transient Ischemic attacks. This is due to distal embolization Relative risk of Invasive Breast Cancer based on Pathological
of platelet thrombi. examination of Benign Breast disease:
•• The procedure involves opening the carotid artery at
stenosis level and closing it with a patch. No risk Adenosis, Sclerosing, Apocrine
metaplasia, Duct ectasia, Cysts,
Indications: Fibrosis, Hyperplasia*, Mastitis,
•• 70% or greater carotid stenosis and Squamous metaplasia
•• Ipsilateral Amaurosis Fugax or Mono ocular Blindness Slightly increased risk Moderate or Florid
•• Contralateral facial paralysis or paresthesia (1.5 -2 times) Hyperplasia**
•• Arm/ leg Paralysis or Paresthesia Papilloma with Fibrovascular
•• Hemianopia core
•• Dysphasia
•• Sensory or Visual inattention/neglect 5 times risk Atypical Hyperplasia
No sufficient data Solitary papilloma of
239. Ans. (b) C4b Lactiferous sinus
Ref: Surgery Sixer 4th Edition Page 837 Radial scar lesion
Clinical Classification
244. Ans. (b) 2, 3, and 4
•• C0- No signs of Venous Disease
•• C1- Telangiectasia (<1 mm Diameter)/Reticular Veins (1– Ref: Bailey and Love 27th Edition Page 873
2.9 mm Diameter) •• Paget’s disease represents the superficial manifestation of an
•• C2- Varicose Vein (>3 mm Diameter) underlying malignant lesion in Breast.
•• C3- Edema •• Nipple Eczema Should be Biopsied.
•• C4-a- Pigmentation, Eczema, b-Lipodermatosclerosis or •• Large Ovoid cells with abundant clear pale cytoplasm in
atrophie blanche Malpighian layer known as Paget cells seen
•• C5- Healed venous ulcer
•• C6- Active venous ulcer 245. Ans. (a) Gastric outlet obstruction
Ref: Bailey and Love 27th Edition Page 1130)
240. Ans. (a) Early-skilled endoscopy is must
•• GOO has a characteristic vomit- Unpleasant with total lack
Ref: Bailey and Love 27th Edition page 1075) of Bile with undigested food particles.
•• Key to management is Early Endoscopy by experienced •• H/o Loss of weight and dehydration+
Endoscopist.
•• Elective esophageal resection delayed for at least 3 months 246. Ans. (b) Crohn’s Disease
unless there is any perforation or Bleeding. Ref: Bailey and Love 27th Edition Page 1241
•• Broad spectrum antibiotics and steroids are not of any use. •• This is a classical case of Crohn’s disease which has-
•• NG tube is usually Contraindicated. Diarrhoea, abdominal pain and multiple perianal sinuses.
•• In Ulcerative colitis, there won’t be Sinus in Anal region.
241. Ans. (b) 2,3 and 4
Ref: Bailey and Love 27th Edition Page 783 247. Ans. (b) CTP B
Salivary gland tumors: Ref: Surgery Sixer 4th Edition Page 592
Incidence of malignant tumors in each gland: •• No Encephalopathy- 1 point
•• Parotid- 10-20% of all tumors •• No ascites- 1 point
•• Submandibular – 50% of all tumors •• > 5mg Bilirubin- 3 points
•• Sublingual- 85% of all tumors •• >3.9 gms Albumin- 1 point
Bailey says- In parotid 80-90% and in Submandibular 60-70% •• INR (Up to 1.7)- 1 point
are Benign tumors. Total points- 7 points. Hence Child Turcotte Pugh Score is
CTP- B
242. Ans. (c) Granulomatous thyroiditis
Grading of liver failure
Ref: Surgery Sixer 4th Edition Page 226
Clinical** 1 2 33
•• This is a case of Subacute or de Quervain’s or Post Viral
Granulomatous inflammatory thyroiditis- Usually resolves Encephalopathy None 1 or 2 3 or 4
itself. Ascites None Mild Moderate
•• High T 4 level and Low RAIU are classical features. Bilirubin (mg/dL) 1–2 mg/dL 2.1–3 mg/dL ≥ 3.1 mg/dL
243. Ans. (a) Hyperplasia Albumin (gm/dL) ≥ 3.5 gm/dL 2.8–3.4 gm/dL ≥ 2.7 mg/dL
Ref: Bailey and Love Table 27 Edition Page 871
th Prothrombin 1–6 4.1–6 ≥6
time (increase in
second)
LXXVIII
Latest Questions Paper 2020-2019
LXXIX
Surgery Sixer for NBE
259. Ans. (c) Open sutures and cleaning of wound 261. Ans. (d) Hyponatremia
Ref Bailey and Love 27th Edition Page 47 Ref: Surgery Sixer 4th Edition Page 5
•• This is a case of Surgical Site infection- Southampton Refeeding Syndrome
Grading system IV with pus projecting.
•• Characterised by severe fluid and electrolyte shifts in
•• Immediate treatment is to let out the pus by opening the
malnourished patients undergoing refeeding*
wound
•• It can occur with Enteral and Parenteral nutrition (MC with
260. Ans. (d) Experiment TPN)
•• Lab values: Hypophosphatemia, Hypocalcemia, Hypo-
Ref: Bailey and Love 27th Edition Page 170, Chapter 12
magnesemia*
This is taken from the Chapter 12- Surgical Ethics which deals
with 262. Ans. (c) Septic shock
•• Respect Autonomy Ref: Bailey and Love 27th Edition Page 14
•• Informed Consent
•• Capillary Refill time is prolonged in hypovolemic and other
•• Practical application
forms of shock.
•• Matters of Life and death
•• Short Capillary filling time is seen in Septic shock and early
•• Confidentiality
stages of all shock.
•• Research
•• Maintaining standards of Excellence
LXXX